All Exams  >   GMAT  >   Practice Questions for GMAT  >   All Questions

All questions of Verbal Reasoning (Level-wise Complete Tests) for GMAT Exam

Some legislators refuse to commit public funds for new scientific research if they cannot be assured that the research will contribute to the public welfare. Such a position ignores the lessons of experience. Many important contributions to the public welfare that resulted from scientific research were never predicted as potential outcomes of that research. Suppose that a scientist in the early twentieth century had applied for public funds to study molds: who would have predicted that such research would lead to the discovery of antibiotics—one of the greatest contributions ever made to the public welfare?

Which one of the following most accurately expresses the main point of the argument?
  • a)
    The committal of public funds for new scientific research will ensure that the public welfare will be enhanced.
  • b)
    If it were possible to predict the general outcome of a new scientific research effort, then legislators would not refuse to commit public funds for that effort.
  • c)
    Scientific discoveries that have contributed to the public welfare would have occurred sooner if public funds had been committed to the research that generated those discoveries.
  • d)
    In order to ensure that scientific research is directed toward contributing to the public welfare, legislators must commit public funds to new scientific research.
  • e)
    Lack of guarantees that new scientific research will contribute to the public welfare is not sufficient reason for legislators to refuse to commit public funds to new scientific research.
Correct answer is option 'E'. Can you explain this answer?

Rahul Kapoor answered
(A) The committal of public funds for new scientific research will ensure that the public welfare will be enhanced. This option does not accurately express the main point of the argument. The argument does not claim that committing public funds for scientific research will ensure enhancement of the public welfare. Instead, it argues against the refusal to commit funds based on the lack of assurance of specific outcomes.
(B) If it were possible to predict the general outcome of a new scientific research effort, then legislators would not refuse to commit public funds for that effort. This option does not accurately express the main point of the argument. The argument acknowledges that predicting the specific outcomes of scientific research is difficult and argues against refusal to fund research based on this lack of predictability. However, the argument does not directly address the possibility of predicting general outcomes.
(C) Scientific discoveries that have contributed to the public welfare would have occurred sooner if public funds had been committed to the research that generated those discoveries. This option does not accurately express the main point of the argument. The argument does not focus on the timing of scientific discoveries or make a claim about the acceleration of these discoveries with public funding. Instead, it emphasizes the unpredictability of specific outcomes and argues against the refusal to fund research based on this unpredictability.
(D) In order to ensure that scientific research is directed toward contributing to the public welfare, legislators must commit public funds to new scientific research. This option does not accurately express the main point of the argument. While the argument advocates for the commitment of public funds to scientific research despite the lack of guarantees, it does not claim that this commitment is necessary to ensure that research is directed towards the public welfare. The argument emphasizes the serendipitous nature of scientific discoveries.
(E) Lack of guarantees that new scientific research will contribute to the public welfare is not sufficient reason for legislators to refuse to commit public funds to new scientific research. This option accurately expresses the main point of the argument. The argument challenges the position of legislators who refuse to allocate public funds for scientific research due to the lack of assurance of specific outcomes. It argues that this lack of guarantees should not be a sufficient reason to deny funding, as unexpected and significant contributions to the public welfare can emerge from scientific research.
Therefore, option (E) is the most accurate expression of the main point of the argument.

Protein synthesis begins when the gene encoding a protein is activated. The gene’s sequence of nucleotides is transcribed into a molecule of messenger RNA (mRNA), which reproduces the information contained in that sequence. Transported outside the nucleus to the cytoplasm, the mRNA is translated into the protein it encodes by an organelle known as a ribosome, which strings together amino acids in the order specified by the sequence of elements in the mRNA molecule. Since the amount of mRNA in a cell determines the amount of the corresponding protein, factors affecting the abundance of mRNA’s play a major part in the normal functioning of a cell by appropriately regulating protein synthesis. For example, an excess of certain proteins can cause cells to proliferate abnormally and become cancerous; a lack of the protein insulin results in diabetes.

Biologists once assumed that the variable rates at which cells synthesize different mRNA’s determine the quantities of mRNA’s and their corresponding proteins in a cell. However, recent investigations have shown that the concentrations of most mRNA’s correlate best, not with their synthesis rate, but rather with the equally variable rates at which cells degrade the different mRNA’s in their cytoplasm. If a cell degrades both a rapidly and a slowly synthesized mRNA slowly, both mRNA’s will accumulate to high levels.

An important example of this phenomenon is the development of red blood cells from their unspecialized parent cells in bone marrow. For red blood cells to accumulate sufficient concentrations of hemoglobin (which transports oxygen) to carry out their main function, the cells’ parent cells must simultaneously produce more of the constituent proteins of hemoglobin and less of most other proteins. To do this, the parent cells halt synthesis of non-hemoglobin mRNA’s in the nucleus and rapidly degrade copies of the non-hemoglobin mRNA’s remaining in the cytoplasm. Halting synthesis of mRNA alone would not affect the quantities of proteins synthesized by the mRNA’s still existing in the cytoplasm. Biologists now believe that most cells can regulate protein production most efficiently by varying both mRNA synthesis and degradation, as developing red cells do, rather than by just varying one or the other.
The accumulation of concentrations of hemoglobin in red blood cells is mentioned in the passage as an example of which of the following?
  • a)
    The effectiveness of simultaneous variation of the rates of synthesis and degradation of mRNA
  • b)
    The role of the ribosome in enabling a parent cell to develop properly into a more specialized form
  • c)
    The importance of activating the genes for particular proteins at the correct moment
  • d)
    The abnormal proliferation of a protein that threatens to make the cell cancerous
  • e)
    The kind of evidence that biologists relied on for support of a view of mRNA synthesis that is now considered obsolete
Correct answer is option 'A'. Can you explain this answer?

Accumulation of Hemoglobin in Red Blood Cells as an Example of Simultaneous Variation of mRNA Synthesis and Degradation
Understanding the Example:
- The passage mentions the accumulation of hemoglobin in red blood cells as an example.
- This example illustrates how cells regulate protein production efficiently.
Simultaneous Variation of Rates:
- Red blood cells need to accumulate sufficient hemoglobin to carry out their main function.
- To achieve this, the parent cells halt synthesis of non-hemoglobin mRNA in the nucleus.
- They also rapidly degrade non-hemoglobin mRNA copies remaining in the cytoplasm.
Efficiency in Protein Production Regulation:
- Biologists now believe that cells regulate protein production most efficiently by varying both mRNA synthesis and degradation.
- This process ensures that the necessary proteins, such as hemoglobin in red blood cells, are produced in the required quantities.
- By simultaneously adjusting mRNA synthesis and degradation rates, cells can achieve optimal protein levels for specific functions.
Conclusion:
- The example of hemoglobin accumulation in red blood cells demonstrates the effectiveness of simultaneous variation of mRNA synthesis and degradation.
- This approach allows cells to precisely regulate protein production based on their functional requirements.

Protein synthesis begins when the gene encoding a protein is activated. The gene’s sequence of nucleotides is transcribed into a molecule of messenger RNA (mRNA), which reproduces the information contained in that sequence. Transported outside the nucleus to the cytoplasm, the mRNA is translated into the protein it encodes by an organelle known as a ribosome, which strings together amino acids in the order specified by the sequence of elements in the mRNA molecule. Since the amount of mRNA in a cell determines the amount of the corresponding protein, factors affecting the abundance of mRNA’s play a major part in the normal functioning of a cell by appropriately regulating protein synthesis. For example, an excess of certain proteins can cause cells to proliferate abnormally and become cancerous; a lack of the protein insulin results in diabetes.

Biologists once assumed that the variable rates at which cells synthesize different mRNA’s determine the quantities of mRNA’s and their corresponding proteins in a cell. However, recent investigations have shown that the concentrations of most mRNA’s correlate best, not with their synthesis rate, but rather with the equally variable rates at which cells degrade the different mRNA’s in their cytoplasm. If a cell degrades both a rapidly and a slowly synthesized mRNA slowly, both mRNA’s will accumulate to high levels.

An important example of this phenomenon is the development of red blood cells from their unspecialized parent cells in bone marrow. For red blood cells to accumulate sufficient concentrations of hemoglobin (which transports oxygen) to carry out their main function, the cells’ parent cells must simultaneously produce more of the constituent proteins of hemoglobin and less of most other proteins. To do this, the parent cells halt synthesis of non-hemoglobin mRNA’s in the nucleus and rapidly degrade copies of the non-hemoglobin mRNA’s remaining in the cytoplasm. Halting synthesis of mRNA alone would not affect the quantities of proteins synthesized by the mRNA’s still existing in the cytoplasm. Biologists now believe that most cells can regulate protein production most efficiently by varying both mRNA synthesis and degradation, as developing red cells do, rather than by just varying one or the other.
The passage is primarily concerned with discussing the
  • a)
    influence of mRNA concentrations on the development of red blood cells
  • b)
    role of the synthesis and degradation of mRNA in cell functioning
  • c)
    mechanism by which genes are transcribed into mRNA
  • d)
    differences in mRNA concentrations in cell nuclei and in the cytoplasm
  • e)
    way in which mRNA synthesis contributes to the onset of diabetes
Correct answer is option 'B'. Can you explain this answer?

Niti Choudhury answered
Overview of the Passage
The passage primarily discusses how mRNA synthesis and degradation influence protein production in cells, emphasizing their role in cellular functioning.
Key Focus: Role of mRNA
- The passage highlights that the regulation of protein synthesis is not solely determined by the rate of mRNA synthesis, but significantly by the degradation rates of mRNA.
- It mentions that mRNA concentrations correlate more with degradation rates than with synthesis rates.
Red Blood Cell Development Example
- The development of red blood cells is used as a pivotal example:
- Parent cells in bone marrow increase hemoglobin production by halting non-hemoglobin mRNA synthesis.
- They actively degrade existing non-hemoglobin mRNA to ensure that the focus remains on producing hemoglobin.
Conclusion of Findings
- The passage concludes that efficient regulation of protein production involves a balance of both mRNA synthesis and degradation.
- This dual approach allows cells to manage protein levels effectively, which is crucial for normal functions and preventing conditions such as cancer or diabetes.
Final Insight
In summary, the passage emphasizes that understanding the interplay of mRNA synthesis and degradation is vital for comprehending cellular functionality, making option 'B' the correct answer.

The simple facts are these: the number of people killed each year by grizzly bears is about the same as the number of people killed by lightning on golf courses. And the number of people killed by lightning on golf courses each year is about the same as the number of people electrocuted by electric blenders. All the horrible myths and gruesome stories aside, therefore, a grizzly bear is in fact about as dangerous as an electric blender or a game of golf.

Which one of the following is an assumption that the author relies upon in the passage?
  • a)
    Most incidents involving grizzly bears are fatal.
  • b)
    Grizzly bears are no longer the danger they once were.
  • c)
    The number of fatalities per year is an adequate indication of something’s dangerousness.
  • d)
    A golf course is a particularly dangerous place to be in a thunderstorm.
  • e)
    Something is dangerous only if it results in death in the majority of cases.
Correct answer is option 'C'. Can you explain this answer?

Rahul Kapoor answered
A. Most incidents involving grizzly bears are fatal. This option suggests that the majority of incidents involving grizzly bears result in fatalities. However, the passage does not provide information or make claims about the fatality rate of incidents involving grizzly bears. Therefore, this option is not the assumption the author relies upon.
B. Grizzly bears are no longer the danger they once were. This option implies that grizzly bears are less dangerous now compared to the past. However, the passage does not discuss any changes in the danger level of grizzly bears over time. Therefore, this option is not the assumption the author relies upon.
C. The number of fatalities per year is an adequate indication of something's dangerousness. This option aligns with the argument made in the passage. The passage compares the number of fatalities caused by different entities (grizzly bears, lightning on golf courses, and electric blenders) to suggest that they are all similarly dangerous. This comparison relies on the assumption that the number of fatalities per year is a suitable indicator of something's dangerousness. If this assumption were false, and the number of fatalities alone was not sufficient to determine dangerousness, it would weaken the argument's conclusion. Therefore, this is the assumption the author relies upon.
D. A golf course is a particularly dangerous place to be in a thunderstorm. This option focuses on the danger of being on a golf course during a thunderstorm. However, the passage does not directly address the danger level of golf courses during thunderstorms or make any specific claims about the danger of being on a golf course in such conditions. Therefore, this option is not the assumption the author relies upon.
E. Something is dangerous only if it results in death in the majority of cases. This option suggests that something can be considered dangerous only if it results in death in the majority of cases. The passage does not explicitly state or imply this condition for determining dangerousness. Instead, it compares the number of fatalities caused by different entities to argue for their similar levels of danger. Therefore, this option is not the assumption the author relies upon.
In summary, option C is the assumption the author relies upon, which is that the number of fatalities per year is an adequate indication of something's dangerousness.

Despite the best efforts of astronomers, no one has yet succeeded in exchanging messages with intelligent life on other planets or in other solar systems. In fact, no one has even managed to prove that any kind of extraterrestrial life exists. Thus, there is clearly no intelligent life anywhere but on Earth.
The argument’s reasoning is flawed because the argument
  • a)
    fails to consider that there might be extraterrestrial forms of intelligence that are not living beings
  • b)
    confuses an absence of evidence for a hypothesis with the existence of evidence against the hypothesis
  • c)
    interprets a disagreement over a scientific theory as a disproof of that theory
  • d)
    makes an inference that relies on the vagueness of the term “life”
  • e)
    relies on a weak analogy rather than on evidence to draw a conclusion
Correct answer is option 'B'. Can you explain this answer?

Nayanika Bajaj answered
In the statement is flawed and based on several assumptions that are not necessarily true.

Firstly, the statement assumes that the lack of success in exchanging messages with intelligent life or proving the existence of extraterrestrial life implies its nonexistence. However, absence of evidence is not evidence of absence. Just because we haven't found evidence yet, doesn't mean that intelligent life doesn't exist elsewhere in the universe.

Secondly, the statement assumes that astronomers' efforts are exhaustive and representative of all possible methods of detecting or contacting extraterrestrial life. However, our current technological capabilities and methods of searching for extraterrestrial life are limited. There might be forms of life or communication that we are unable to detect or understand at the moment.

Furthermore, the statement assumes that intelligent life can only exist in a way that is recognizable or understandable by humans. It is possible that intelligent life on other planets might have completely different forms of communication or technology that we are currently unable to comprehend.

Lastly, the statement assumes that Earth is the only planet capable of supporting intelligent life. Given the vastness of the universe and the countless number of potentially habitable planets, it is statistically unlikely that Earth is the only planet where intelligent life exists.

In conclusion, the argument that there is no intelligent life anywhere but on Earth is based on flawed assumptions and lacks sufficient evidence. It is premature to make such a definitive statement until further exploration and technological advancements are made.

Using broad-spectrum weed killers on weeds that are competing with crops for sunlight, water, and nutrients presents a difficulty: how to keep the crop from being killed along with the weeds. For at least some food crops, specially treated seed that produces plants resistant to weed killers is under development. This resistance wears off as the plants mature. Therefore, the special seed treatment will be especially useful for plants that _______.
Which of the following most logically completes the argument below?
  • a)
    produce their crop over an extended period of time, as summer squash does
  • b)
    produce large seeds that are easy to treat individually, as corn and beans do
  • c)
    provide, as they approach maturity, shade dense enough to keep weeds from growing
  • d)
    are typically grown in large tracts devoted to a single crop
  • e)
    are cultivated specifically for the seed they produce rather than for their leaves or roots
Correct answer is option 'C'. Can you explain this answer?

Rahul Kapoor answered
The argument discusses the difficulty of using broad-spectrum weed killers on crops because they can kill the crops along with the weeds. It suggests that specially treated seed, which provides resistance to weed killers but wears off as the plants mature, can be a solution. The missing information is about which plants the special seed treatment will be especially useful for. We need to find an option that logically completes the argument. Let's evaluate each option:
(A) produce their crop over an extended period of time, as summer squash does. This option is not directly related to the issue of weed control. The fact that a crop produces its crop over an extended period of time does not address the effectiveness of the special seed treatment for weed control.
(B) produce large seeds that are easy to treat individually, as corn and beans do. This option is also not directly related to the issue of weed control. The ease of treating individual seeds does not address the effectiveness of the special seed treatment for weed control.
(C) provide, as they approach maturity, shade dense enough to keep weeds from growing. This option completes the argument logically. If the crops provide dense shade as they approach maturity, it suggests that the crops themselves can suppress weed growth, reducing the reliance on weed killers. Therefore, the special seed treatment would be especially useful for plants that provide shade dense enough to keep weeds from growing.
(D) are typically grown in large tracts devoted to a single crop. This option is not directly related to the issue of weed control. The size of the crop tracts does not address the effectiveness of the special seed treatment for weed control.
(E) are cultivated specifically for the seed they produce rather than for their leaves or roots. This option is also not directly related to the issue of weed control. The purpose of cultivating the plants for their seeds does not address the effectiveness of the special seed treatment for weed control.
Based on the analysis, option (C) most logically completes the argument by providing a characteristic of plants that would make the special seed treatment especially useful for weed control.

Hea Sook: One should not readily believe urban legends. Most legends are propagated because the moral lesson underlying them supports a political agenda. People will repeat a tale if it fits their purpose. They may not deliberately spread untruths, but neither are they particularly motivated to investigate deeply to determine if the tale they are telling is true.
Kayla: But people would not repeat stories that they did not believe were true. Therefore, one can safely assume that if a story has been repeated by enough people then it is more likely to be true.
Kayla's reply is most vulnerable to the criticism that it
  • a)
    does not specify how many people need to repeat a story before someone is justified believing it
  • b)
    overstates the significance of political agendas in the retelling of stories
  • c)
    fails to address the claim that people will not verify the truth of a story that fits their purpose
  • d)
    implicitly supports the claim that the people repeating legends are not deliberately spreading untruths
  • e)
    cannot distinguish people's motivations for repeating urban legends from their motivations for repeating other types of story
Correct answer is option 'C'. Can you explain this answer?

A. does not specify how many people need to repeat a story before someone is justified believing it: This criticism is not directly related to Kayla's statement. Kayla's argument is about the likelihood of a story being true based on the number of people repeating it, rather than specifying a specific number of people required to believe it.
B. overstates the significance of political agendas in the retelling of stories: This option does not address the vulnerability of Kayla's statement. It focuses on the extent of political agendas in the retelling of stories, which is not the main point of contention in the conversation.
C. fails to address the claim that people will not verify the truth of a story that fits their purpose: This option correctly identifies the vulnerability in Kayla's statement. Hea Sook argues that people are not motivated to investigate deeply to verify the truth of a story if it aligns with their purpose or agenda. Kayla's response does not address this claim and assumes that people would not repeat stories they do not believe to be true.
D. implicitly supports the claim that the people repeating legends are not deliberately spreading untruths: This option does not directly address the vulnerability of Kayla's statement. It focuses on the claim about people deliberately spreading untruths, which is not the central point of disagreement in the conversation.
E. cannot distinguish people's motivations for repeating urban legends from their motivations for repeating other types of stories: This option does not address the vulnerability in Kayla's statement. It focuses on distinguishing motivations for repeating urban legends from other types of stories, which is not relevant to the argument being discussed.
Therefore, the correct answer is C. fails to address the claim that people will not verify the truth of a story that fits their purpose.

A detailed lab analysis of a meteorite recently discovered in Antarctica revealed that the meteorite has geological characteristics common to the planet Mars. To date, scientists have not found these characteristics anywhere other than on Mars.using a technique called "acid-etching" scientist found that the meteorite contained fossilized remains of single-cell life forms.
The statement above, if true, best support which of the following as a conclusion?
  • a)
    The fossilized remains indicate that life exists on Mar
  • b)
    The scientists have evidence to support a hypothesis that the meteorite came from mars.
  • c)
    The meteorite represents a substantial step forward in human knowledge of the development of life in the solar system.
  • d)
    undiscovered meteorites currently exist in Antarctica
  • e)
    "Acid - etching" is necessary to confirm the existence of fossilized remains in meteorites.
Correct answer is option 'B'. Can you explain this answer?

Rahul Kapoor answered
b) The scientists have evidence to support a hypothesis that the meteorite came from Mars.
The given information states that a meteorite discovered in Antarctica has geological characteristics common to Mars and contains fossilized remains of single-cell life forms. This suggests that the meteorite originated from Mars, providing evidence to support the hypothesis that it came from the planet. Therefore, option (b) is the most appropriate conclusion based on the provided information.
Options (a), (c), (d), and (e) are not directly supported by the given information. The statement does not provide sufficient evidence to conclude that life exists on Mars (option a), make a broad claim about human knowledge of life development in the solar system (option c), assert the existence of undiscovered meteorites in Antarctica (option d), or state that "acid-etching" is necessary to confirm the existence of fossilized remains in meteorites (option e).

Over the past twenty years in the U.S., the average number of hours per week that people spend at work has increased from approximately 41 hours to nearly 52 hours. It is thought that this change has played an important role in the corresponding increase in average body mass index for working Americans over the same period. The increased time at work does not allow people as much time to exercise and engage in healthy activities that help reduce weight.
Which of the following questions would be most useful to answer in determining whether the increased workweek is an important cause of the increase in average body mass index over the past twenty years?
  • a)
    Do more employers offer healthy eating options in their onsite cafeterias today compared to twenty years ago?
  • b)
    Do more employers subsidize gym and health club memberships for their employees today compared to twenty years ago?
  • c)
    What percentage of employees use their free time to exercise and engage in healthy activities today compared to twenty years ago?
  • d)
    Did employees exercise and engage in healthy activities with the additional time when they were not at work twenty years ago?
  • e)
    What factors other than exercise and engaging in healthy activities are important for weight loss?
Correct answer is option 'D'. Can you explain this answer?

Understanding the Impact of Increased Work Hours
To determine whether the increase in work hours is a significant cause of the rise in average body mass index (BMI) among Americans, it’s crucial to consider the context of lifestyle changes over the past twenty years.
Key Consideration: Historical Context of Free Time
- The question “Did employees exercise and engage in healthy activities with the additional time when they were not at work twenty years ago?” focuses on understanding past behaviors.
- This inquiry is vital because it compares the lifestyle choices of individuals two decades ago with those today.
Reasons Why Option D is Crucial
- Behavioral Patterns: If employees were already less active in their free time twenty years ago, the additional hours at work may not have drastically changed their activity levels.
- Causal Link: Establishing a historical baseline of physical activity helps determine if the increased work hours directly correlate to a decline in exercise and an increase in BMI.
- Comprehensive Analysis: It allows for a broader understanding of how work-life balance affects health, rather than merely attributing weight gain to longer hours without considering past behavior.
Conclusion
By examining past employee behaviors regarding exercise and healthy activities, we can better understand if the increase in work hours is a significant factor contributing to the rise in average BMI. This understanding can inform future health initiatives and workplace policies.

Climatic conditions are delicately adjusted to the composition of the Earth’s atmosphere. If there were a change in the atmosphere—for example, in the relative proportions of atmospheric gases—the climate would probably change also. A slight increase in water vapor, for instance, would increase the heat-retaining capacity of the atmosphere and would lead to a rise in global temperatures. In contrast, a large increase in water vapor would increase the thickness and extent of the cloud layer, reducing the amount of solar energy reaching the Earth’s surface.
The level of carbon dioxide, CO2, in the atmosphere has an important effect on climatic change. Most of the Earth’s incoming energy is short-wavelength radiation, which tends to pass through atmospheric CO2 easily. The Earth, however, reradiates much of the received energy as long-wavelength radiation, which CO2 absorbs and then remits toward the Earth. This phenomenon, known as the greenhouse effect, can result in an increase in the surface temperature of a planet. An extreme example of the effect is shown by Venus, a planet covered by heavy clouds composed mostly of CO2, whose surface temperatures have been measured at 430℃. If the CO2 content of the atmosphere is reduced, the temperature falls. According to one respectable theory, if the atmospheric CO2 concentration were halved, the Earth would become completely covered with ice. Another equally respectable theory, however, states that a halving of the CO2 concentration would lead only to a reduction in global temperatures of 3℃.
If, because of an increase in forest fires or volcanic activity, the CO2 content of the atmosphere increased, a warmer climate would be produced. Plant growth, which relies on both the warmth and the availability of CO2 would probably increase. As a consequence, plants would use more and more CO2. Eventually CO2 levels would diminish and the climate, in turn, would become cooler. With reduced temperatures many plants would die; CO2 would thereby be returned to the atmosphere and gradually the temperature would rise again. Thus, if this process occurred, there might be a long-term oscillation in the amount of CO2 present in the atmosphere, with regular temperature increases and decreases of a set magnitude.
Some climatologists argue that the burning of fossil fuels has raised the level of CO2 in the atmosphere and has caused a global temperature increase of at least 1℃. But a supposed global temperature rise of 1℃ may in reality be only several regional temperature increases, restricted to areas where there are many meteorological stations and caused simply by shifts in the pattern of atmospheric circulation. Other areas, for example the Southern Hemisphere oceanic zone, may be experiencing an equivalent temperature decrease that is unrecognized because of the shortage of meteorological recording stations.
The passage supplies information for answering which of the following questions?
  • a)
    Why are projections of the effects of changes in water vapor levels on the climate so inaccurate?
  • b)
    What are the steps in the process that takes place as CO2 absorbs long-wavelength radiation?
  • c)
    How might our understanding of the greenhouse effect be improved if the burning of fossil fuels were decreased?
  • d)
    What might cause a series of regular increases and decreases in the amount of CO2 in the atmosphere?
  • e)
    Why are there fewer meteorological recording stations in the Southern Hemisphere oceanic zone than elsewhere?
Correct answer is option 'D'. Can you explain this answer?

Understanding the Oscillation of CO2 Levels
The passage highlights a potential mechanism that could lead to a series of regular increases and decreases in atmospheric CO2 levels. This is primarily due to the interplay between climate change, CO2 concentration, and plant growth.
Causes of CO2 Fluctuation
- Increased CO2 Sources: Events like forest fires or volcanic eruptions can elevate CO2 levels in the atmosphere, contributing to a warmer climate.
- Plant Growth Dynamics: Warmer temperatures and higher CO2 levels can enhance plant growth. Plants absorb CO2, which leads to a gradual decrease in atmospheric CO2 as they thrive.
- Temperature Feedback Loop: If CO2 levels drop significantly due to extensive plant growth, the climate could cool, leading to plant die-off. This die-off would return CO2 to the atmosphere, potentially causing temperatures to rise again.
Long-Term Oscillation
- Regular Temperature Changes: This cycle may create a pattern of oscillation in CO2 levels, marked by regular temperature increases and decreases. The passage implies that such fluctuations could occur with a specific magnitude over time.
Conclusion
In summary, the long-term oscillation of CO2 and its corresponding temperature changes is driven by natural feedback mechanisms between atmospheric CO2 levels, climate conditions, and plant growth. This cyclical process is fundamental in understanding how CO2 concentrations might fluctuate over time, making option 'D' the correct answer to the question regarding the causes of these regular increases and decreases in atmospheric CO2 levels.

Protein synthesis begins when the gene encoding a protein is activated. The gene’s sequence of nucleotides is transcribed into a molecule of messenger RNA (mRNA), which reproduces the information contained in that sequence. Transported outside the nucleus to the cytoplasm, the mRNA is translated into the protein it encodes by an organelle known as a ribosome, which strings together amino acids in the order specified by the sequence of elements in the mRNA molecule. Since the amount of mRNA in a cell determines the amount of the corresponding protein, factors affecting the abundance of mRNA’s play a major part in the normal functioning of a cell by appropriately regulating protein synthesis. For example, an excess of certain proteins can cause cells to proliferate abnormally and become cancerous; a lack of the protein insulin results in diabetes.

Biologists once assumed that the variable rates at which cells synthesize different mRNA’s determine the quantities of mRNA’s and their corresponding proteins in a cell. However, recent investigations have shown that the concentrations of most mRNA’s correlate best, not with their synthesis rate, but rather with the equally variable rates at which cells degrade the different mRNA’s in their cytoplasm. If a cell degrades both a rapidly and a slowly synthesized mRNA slowly, both mRNA’s will accumulate to high levels.

An important example of this phenomenon is the development of red blood cells from their unspecialized parent cells in bone marrow. For red blood cells to accumulate sufficient concentrations of hemoglobin (which transports oxygen) to carry out their main function, the cells’ parent cells must simultaneously produce more of the constituent proteins of hemoglobin and less of most other proteins. To do this, the parent cells halt synthesis of non-hemoglobin mRNA’s in the nucleus and rapidly degrade copies of the non-hemoglobin mRNA’s remaining in the cytoplasm. Halting synthesis of mRNA alone would not affect the quantities of proteins synthesized by the mRNA’s still existing in the cytoplasm. Biologists now believe that most cells can regulate protein production most efficiently by varying both mRNA synthesis and degradation, as developing red cells do, rather than by just varying one or the other.
Which of the following best describes the relationship between the second and third paragraphs of the passage?
  • a)
    The second paragraph presents arguments in support of a new theory and the third paragraph presents arguments against that same theory.
  • b)
    The second paragraph describes a traditional view and the third paragraph describes the view that has replaced it on the basis of recent investigations.
  • c)
    The third paragraph describes a specific case of a phenomenon that is described generally in the second paragraph.
  • d)
    The third paragraph describes an investigation that was undertaken to resolve problems raised by phenomena described in the second paragraph.
  • e)
    Both paragraphs describe in detail specific examples of the phenomenon that is introduced in the first paragraph.
Correct answer is option 'C'. Can you explain this answer?

Niharika Sen answered
Understanding the Relationship Between the Paragraphs
The connection between the second and third paragraphs is best captured by option 'C'. Here’s why:
Traditional View vs. Specific Example
- The second paragraph outlines a general phenomenon regarding mRNA synthesis and degradation, emphasizing that the balance of these processes influences the amount of proteins produced in cells.
- It discusses how traditional beliefs focused primarily on the rates of mRNA synthesis, whereas recent studies indicate that degradation rates play a crucial role in determining mRNA levels.
Specific Case of Red Blood Cell Development
- The third paragraph provides a concrete example of this phenomenon by discussing the specific case of red blood cell (RBC) development in bone marrow.
- It illustrates how parent cells of RBCs manage the synthesis and degradation of mRNA to ensure adequate hemoglobin production, highlighting the practical application of the concepts introduced in the second paragraph.
Conclusion
- Thus, while the second paragraph sets the stage with a broad overview of the mRNA dynamics, the third paragraph zooms in on a specific instance (RBC development) that exemplifies the broader phenomenon discussed earlier.
This makes option 'C' the most accurate description of the relationship between the two paragraphs, as it captures the transition from a general principle to a specific illustration.

The amount of electricity consumed in Millville on any day in August is directly proportional to peak humidity on that day. Since the average peak humidity this August was three points higher than the average peak humidity last August, it follows that more energy was consumed in Millville this August than last August.
Which one of the following arguments has a pattern of reasoning most similar to the one in the argument above?
  • a)
    The amount of art supplies used in any of the Aesthetic Institute’s 25 classes is directly proportional to the number of students in that class. Since in these classes the institute enrolled 20 percent more students overall last year than in the previous year, more art supplies were used in the institute’s classes last year than in the previous year.
  • b)
    The number of courses in painting offered by the Aesthetic Institute in any term is directly proportional to the number of students enrolled in the institute in that term. But the institute offers the same number of courses in sculpture each term. Hence, the institute usually offers more courses in painting than in sculpture each term.
  • c)
    The number of new students enrolled at the Aesthetic Institute in any given year is directly proportional to the amount of advertising the institute has done in the previous year. Hence, if the institute seeks to increase its student body it must increase the amount it spends on advertising.
  • d)
    The fees paid by a student at the Aesthetic Institute are directly proportional to the number of classes in which that student enrolls. Since the number of students at the Aesthetic Institute is increasing, it follows that the institute is collecting a greater amount in fees paid by students than it used to.
  • e)
    The number of instructors employed by the Aesthetic Institute in any term is directly proportional to the number of classes offered in that term and also directly proportional to the number of students enrolled at the institute. Thus, the number of classes offered by the institute in any term is directly proportional to the number of students enrolled in that term.
Correct answer is option 'A'. Can you explain this answer?

(A) The amount of art supplies used in any of the Aesthetic Institute’s 25 classes is directly proportional to the number of students in that class. Since in these classes the institute enrolled 20 percent more students overall last year than in the previous year, more art supplies were used in the institute’s classes last year than in the previous year. This argument follows a similar pattern of reasoning. It states that the amount of art supplies used is directly proportional to the number of students and concludes that more supplies were used due to a 20 percent increase in the number of students.
(B) The number of courses in painting offered by the Aesthetic Institute in any term is directly proportional to the number of students enrolled in the institute in that term. But the institute offers the same number of courses in sculpture each term. Hence, the institute usually offers more courses in painting than in sculpture each term. This argument does not follow the same pattern of reasoning. It establishes a relationship between the number of courses in painting and the number of students but concludes that the institute usually offers more courses in painting based on the fact that the number of sculpture courses remains the same. It does not involve a change in one variable leading to a conclusion about the change in another variable.
(C) The number of new students enrolled at the Aesthetic Institute in any given year is directly proportional to the amount of advertising the institute has done in the previous year. Hence, if the institute seeks to increase its student body, it must increase the amount it spends on advertising. This argument establishes a relationship between the number of new students and the amount of advertising, but it does not involve a change in one variable leading to a conclusion about the change in another variable. It does not follow the same pattern of reasoning.
(D) The fees paid by a student at the Aesthetic Institute are directly proportional to the number of classes in which that student enrolls. Since the number of students at the Aesthetic Institute is increasing, it follows that the institute is collecting a greater amount in fees paid by students than it used to. This argument follows a similar pattern of reasoning. It states that fees paid by a student are directly proportional to the number of classes and concludes that the institute is collecting a greater amount in fees due to an increase in the number of students.
(E) The number of instructors employed by the Aesthetic Institute in any term is directly proportional to the number of classes offered in that term and also directly proportional to the number of students enrolled at the institute. Thus, the number of classes offered by the institute in any term is directly proportional to the number of students enrolled in that term. This argument establishes relationships between the number of instructors, the number of classes, and the number of students. It does not involve a change in one variable leading to a conclusion about the change in another variable. It does not follow the same pattern of reasoning.
Based on the evaluation, the argument that has a pattern of reasoning most similar to the one in the given argument is (A) The amount of art supplies used in any of the Aesthetic Institute’s 25 classes is directly proportional to the number of students in that class. Since in these classes, the institute enrolled 20 percent more students overall last year than in the previous year, more art supplies were used in the institute’s classes last year than in the previous year.

According to many analysts, labor-management relations in the United States are undergoing a fundamental change: traditional adversarialism is giving way to a new cooperative relationship between the two sides and even to concessions from labor. These analysts say the twin shocks of nonunion competition in this country and low-cost, high-quality imports from abroad are forcing unions to look more favorably at a variety of management demands: the need for wage restraint and reduced benefits as well as the abolition of “rigid” work rules, seniority rights, and job classifications.
Sophisticated proponents of these new developments cast their observations in a prolabor light. In return for their concessions, they point out, some unions have bargained for profit sharing, retraining rights, and job¬-security guarantees. Unions can also trade concessions for more say on the shop floor, where techniques such as quality circles and quality-of-work-life programs promise workers greater control over their own jobs. Unions may even win a voice in investment and pricing strategy, plant location, and other major corporate policy decisions previously reserved to management.
Opponents of these concessions from labor argue that such concessions do not save jobs, but either prolong the agony of dying plants or finance the plant relocations that employers had intended anyway. Companies make investment decisions to fit their strategic plans and their profit objectives, opponents point out, and labor costs are usually just a small factor in the equation. Moreover, unrestrained by either loyalty to their work force or political or legislative constraints on their mobility, the companies eventually cut and run, concessions or no concessions.
Wage-related concessions have come under particular attack, since opponents believe that high union wages underlay much of the success of United States industry in this century. They point out that a long-standing principle, shared by both management and labor, has been that workers should earn wages that give them the income they need to buy what they make. Moreover, high wages have given workers the buying power to propel the economy forward. If proposals for pay cuts, two-tier wage systems, and subminimum wages for young workers continue to gain credence, opponents believe the U.S. social structure will move toward that of a less-developed nation: a small group of wealthy investors, a sizable but still minority bloc of elite professionals and highly skilled employees, and a huge mass of marginal workers and unskilled laborers. Further, they argue that if unions willingly engage in concession bargaining on the false grounds that labor costs are the source of a company's problems, unions will find themselves competing with Third World pay levels—a competition they cannot win.
 The passage provides information to answer which of the following questions?
  • a)
    What has caused unions to consider wage restraints and reduced benefits?
  • b)
    Why do analysts study United States labor­-management relations?
  • c)
    How do job-security guarantees operate?
  • d)
    Are investment and pricing strategies effective in combating imports?
  • e)
    Do quality circles improve product performance and value?
Correct answer is option 'A'. Can you explain this answer?

EduRev GMAT answered
The passage explains that the twin shocks of nonunion competition within the United States and low-cost, high-quality imports from abroad are pressuring unions to be more open to management demands, including wage restraints and reduced benefits.
This information is clearly addressed in the first paragraph: "These analysts say the twin shocks of nonunion competition in this country and low-cost, high-quality imports from abroad are forcing unions to look more favorably at a variety of management demands: the need for wage restraint and reduced benefits..."
Incorrect Options:
(B) Why do analysts study United States labor-management relations?
The passage does not discuss the reasons analysts study these relations but rather the current state and changes in labor-management relations.
(C) How do job-security guarantees operate?
The passage mentions job-security guarantees but does not provide details on their operation.
(D) Are investment and pricing strategies effective in combating imports?
This topic is not addressed in the passage.
(E) Do quality circles improve product performance and value?
While quality circles are mentioned, their impact on product performance and value is not discussed.

For hot desert locations with access to seawater, a new greenhouse design generates freshwater and cool air. Oriented to the prevailing wind, the front wall of perforated cardboard, moistened by a trickle of seawater pumped in, cools and moistens hot air blowing in. This cool, humidified air accelerates plant growth; little water evaporates from leaves. Though greenhouses normally capture the heat of sunlight, a double-layered roof, the inner layer coated to reflect infrared light outward, allows visible sunlight in but traps solar heat between the two layers. This heated air, drawn down from the roof, then mixes with the greenhouse air as it reaches a second seawater-moistened cardboard wall at the back of the greenhouse. There the air absorbs more moisture before being cooled off again when it meets a seawater-cooled metal wall, which causes moisture in the air to condense. Thus distilled water for irrigating the plants collects.

Q. Which of the following can be inferred about the importance of the seawater-moistened cardboard walls in the greenhouse design?
  • a)
    They serve as the primary source of freshwater for the greenhouse.
  • b)
    They help in maintaining the structure of the greenhouse.
  • c)
    They are crucial for cooling and humidifying the air inside the greenhouse.
  • d)
    They ensure the proper circulation of air within the greenhouse.
  • e)
    They reflect solar heat away from the greenhouse to keep it cool.
Correct answer is option 'C'. Can you explain this answer?

The passage indicates that the seawater-moistened cardboard walls cool and moisten the hot air blowing into the greenhouse and later help the air absorb more moisture before cooling it again to condense moisture. Hence, their main function is to cool and humidify the air inside the greenhouse.

Lawyer: Juries are traditionally given their instructions in convoluted, legalistic language. The verbiage is intended to make the instructions more precise, but greater precision is of little use if most jurors have difficulty understanding the instructions. Since it is more important for jurors to have a basic but adequate understanding of their role than it is for the details of that role to be precisely specified, jury instructions should be formulated in simple, easily comprehensible language.

Each of the following, if true, strengthens the lawyer's argument EXCEPT:
  • a)
    Most jurors are less likely to understand instructions given in convoluted, legalistic language than instructions given in simple, easily comprehensible language.
  • b)
    Most jurors do not have an adequate understanding of their role after being given jury instructions in convoluted, legalistic language.
  • c)
    Jury instructions formulated in simple, easily comprehensible language can adequately describe the role of the jurors.
  • d)
    The details of the role of the jurors cannot be specified with complete precision in simple, easily comprehensible language.
  • e)
    Jurors do not need to know the precise details of their role in order to have an adequate understanding of that role.
Correct answer is option 'D'. Can you explain this answer?

(A) Most jurors are less likely to understand instructions given in convoluted, legalistic language than instructions given in simple, easily comprehensible language. This statement supports the lawyer's argument by highlighting the difficulty jurors have in understanding convoluted language. It strengthens the argument.
(B) Most jurors do not have an adequate understanding of their role after being given jury instructions in convoluted, legalistic language. This statement supports the lawyer's argument by suggesting that convoluted language fails to provide jurors with an adequate understanding of their role. It strengthens the argument.
(C) Jury instructions formulated in simple, easily comprehensible language can adequately describe the role of the jurors. This statement directly supports the lawyer's argument by stating that simple language can adequately describe the jurors' role. It strengthens the argument.
(D) The details of the role of the jurors cannot be specified with complete precision in simple, easily comprehensible language. This statement weakens the lawyer's argument because it suggests that simple language may not be able to convey the precise details of the jurors' role. It does not support the argument.
(E) Jurors do not need to know the precise details of their role in order to have an adequate understanding of that role. This statement supports the lawyer's argument by asserting that a basic understanding of the jurors' role is adequate, without requiring precise details. It strengthens the argument.
Therefore, the answer is (D) The details of the role of the jurors cannot be specified with complete precision in simple, easily comprehensible language.

If the minimum wage increases again, MacDowell’s will have to increase the prices it charges for its products. And if that happens, MacDowell’s has a choice: it can spend more on advertising to attract more customers, or its sales and profitability will decrease. But since the extra advertising costs will simply raise total expenses, increasing those costs will still result in an overall decrease in profitability.
Which one of the following conclusions can be logically drawn from the statements above?
  • a)
    Unless the minimum wage increases, MacDowell’s will continue to remain profitable.
  • b)
    If the minimum wage increases, MacDowell’s will no longer be able to remain profitable.
  • c)
    MacDowell’s will see its profitability increase if the minimum wage does not increase.
  • d)
    MacDowell’s will be unable to maintain its current profitability if the minimum wage increases.
  • e)
    If MacDowell’s sees a reduction in its profitability, that means that the minimum wage has increased.
Correct answer is option 'D'. Can you explain this answer?

A. Unless the minimum wage increases, MacDowell’s will continue to remain profitable. This option cannot be logically drawn from the given statements. The argument discusses the potential impact of a minimum wage increase on MacDowell's profitability, but it does not provide any information about the company's profitability in the absence of a minimum wage increase. Therefore, we cannot conclude that MacDowell's will continue to remain profitable if the minimum wage doesn't increase.
B. If the minimum wage increases, MacDowell’s will no longer be able to remain profitable. This option can be logically drawn from the statements. The argument indicates that if the minimum wage increases, MacDowell's will have to increase its prices. This implies that the company's costs will rise, and if it is unable to offset those costs or maintain its profitability through other means, it may no longer be able to remain profitable.
C. MacDowell’s will see its profitability increase if the minimum wage does not increase. This option cannot be logically drawn from the given statements. The argument focuses on the potential negative impact of a minimum wage increase on MacDowell's profitability but does not provide any information about the profitability in the absence of a minimum wage increase. Therefore, we cannot conclude that MacDowell's profitability will increase if the minimum wage remains unchanged.
D. MacDowell’s will be unable to maintain its current profitability if the minimum wage increases. This option can be logically drawn from the statements. The argument states that if the minimum wage increases, MacDowell's will have to increase its prices, and even if they choose to spend more on advertising, it will still result in an overall decrease in profitability. Therefore, it can be inferred that MacDowell's will be unable to maintain its current profitability if the minimum wage increases.
E. If MacDowell’s sees a reduction in its profitability, that means that the minimum wage has increased. This option cannot be logically drawn from the given statements. The argument discusses the potential consequences of a minimum wage increase on MacDowell's profitability, but it does not establish a causal relationship between a reduction in profitability and an increase in the minimum wage. Other factors could also contribute to a reduction in profitability, so we cannot conclude that a reduction in profitability specifically indicates an increase in the minimum wage.
Based on the explanations above, the correct answer is option D.

Climatic conditions are delicately adjusted to the composition of the Earth’s atmosphere. If there were a change in the atmosphere—for example, in the relative proportions of atmospheric gases—the climate would probably change also. A slight increase in water vapor, for instance, would increase the heat-retaining capacity of the atmosphere and would lead to a rise in global temperatures. In contrast, a large increase in water vapor would increase the thickness and extent of the cloud layer, reducing the amount of solar energy reaching the Earth’s surface.
The level of carbon dioxide, CO2, in the atmosphere has an important effect on climatic change. Most of the Earth’s incoming energy is short-wavelength radiation, which tends to pass through atmospheric CO2 easily. The Earth, however, reradiates much of the received energy as long-wavelength radiation, which CO2 absorbs and then remits toward the Earth. This phenomenon, known as the greenhouse effect, can result in an increase in the surface temperature of a planet. An extreme example of the effect is shown by Venus, a planet covered by heavy clouds composed mostly of CO2, whose surface temperatures have been measured at 430℃. If the CO2 content of the atmosphere is reduced, the temperature falls. According to one respectable theory, if the atmospheric CO2 concentration were halved, the Earth would become completely covered with ice. Another equally respectable theory, however, states that a halving of the CO2 concentration would lead only to a reduction in global temperatures of 3℃.
If, because of an increase in forest fires or volcanic activity, the CO2 content of the atmosphere increased, a warmer climate would be produced. Plant growth, which relies on both the warmth and the availability of CO2 would probably increase. As a consequence, plants would use more and more CO2. Eventually CO2 levels would diminish and the climate, in turn, would become cooler. With reduced temperatures many plants would die; CO2 would thereby be returned to the atmosphere and gradually the temperature would rise again. Thus, if this process occurred, there might be a long-term oscillation in the amount of CO2 present in the atmosphere, with regular temperature increases and decreases of a set magnitude.
Some climatologists argue that the burning of fossil fuels has raised the level of CO2 in the atmosphere and has caused a global temperature increase of at least 1℃. But a supposed global temperature rise of 1℃ may in reality be only several regional temperature increases, restricted to areas where there are many meteorological stations and caused simply by shifts in the pattern of atmospheric circulation. Other areas, for example the Southern Hemisphere oceanic zone, may be experiencing an equivalent temperature decrease that is unrecognized because of the shortage of meteorological recording stations.
The passage suggests that a large decrease in the amount of CO2 in the atmosphere would result in
  • a)
    at least a slight decrease in global temperatures
  • b)
    at the most a slight increase in short-wavelength radiation reaching the Earth
  • c)
    a slight long-term increase in global temperatures
  • d)
    a large long-term increase in the amount of volcanic activity
  • e)
    a slight short-term increase in atmosphere water vapor content
Correct answer is option 'A'. Can you explain this answer?

EduRev GMAT answered
The passage explains that CO2 plays a significant role in the greenhouse effect by absorbing long-wavelength radiation and re-radiating it towards the Earth's surface, thus contributing to warming. It also notes that a reduction in atmospheric CO2 would lead to a decrease in surface temperatures. The passage references theories suggesting that halving the CO2 concentration could lead to dramatic cooling, such as the Earth becoming completely covered with ice or experiencing a 3℃ reduction in global temperatures.
Thus, the passage implies that reducing CO2 levels will have a cooling effect, leading to a decrease in global temperatures. This makes (A) the most accurate answer as it aligns with the described impact of reducing CO2.
The other options do not directly reflect the passage's content:
  • (B) at the most a slight increase in short-wavelength radiation reaching the Earth
    The passage does not discuss an increase in short-wavelength radiation due to CO2 changes; rather, it focuses on the impact on temperature.
  • (C) a slight long-term increase in global temperatures
    A decrease in CO2 would generally lead to a decrease in temperatures, not an increase.
  • (D) a large long-term increase in the amount of volcanic activity
    The passage does not connect CO2 decrease with an increase in volcanic activity.
  • (E) a slight short-term increase in atmospheric water vapor content
    There is no direct link in the passage between a decrease in CO2 and an increase in water vapor content.
Therefore, (A) is the most fitting choice based on the passage's information.

Climatic conditions are delicately adjusted to the composition of the Earth’s atmosphere. If there were a change in the atmosphere—for example, in the relative proportions of atmospheric gases—the climate would probably change also. A slight increase in water vapor, for instance, would increase the heat-retaining capacity of the atmosphere and would lead to a rise in global temperatures. In contrast, a large increase in water vapor would increase the thickness and extent of the cloud layer, reducing the amount of solar energy reaching the Earth’s surface.
The level of carbon dioxide, CO2, in the atmosphere has an important effect on climatic change. Most of the Earth’s incoming energy is short-wavelength radiation, which tends to pass through atmospheric CO2 easily. The Earth, however, reradiates much of the received energy as long-wavelength radiation, which CO2 absorbs and then remits toward the Earth. This phenomenon, known as the greenhouse effect, can result in an increase in the surface temperature of a planet. An extreme example of the effect is shown by Venus, a planet covered by heavy clouds composed mostly of CO2, whose surface temperatures have been measured at 430℃. If the CO2 content of the atmosphere is reduced, the temperature falls. According to one respectable theory, if the atmospheric CO2 concentration were halved, the Earth would become completely covered with ice. Another equally respectable theory, however, states that a halving of the CO2 concentration would lead only to a reduction in global temperatures of 3℃.
If, because of an increase in forest fires or volcanic activity, the CO2 content of the atmosphere increased, a warmer climate would be produced. Plant growth, which relies on both the warmth and the availability of CO2 would probably increase. As a consequence, plants would use more and more CO2. Eventually CO2 levels would diminish and the climate, in turn, would become cooler. With reduced temperatures many plants would die; CO2 would thereby be returned to the atmosphere and gradually the temperature would rise again. Thus, if this process occurred, there might be a long-term oscillation in the amount of CO2 present in the atmosphere, with regular temperature increases and decreases of a set magnitude.
Some climatologists argue that the burning of fossil fuels has raised the level of CO2 in the atmosphere and has caused a global temperature increase of at least 1℃. But a supposed global temperature rise of 1℃ may in reality be only several regional temperature increases, restricted to areas where there are many meteorological stations and caused simply by shifts in the pattern of atmospheric circulation. Other areas, for example the Southern Hemisphere oceanic zone, may be experiencing an equivalent temperature decrease that is unrecognized because of the shortage of meteorological recording stations.
The author is primarily concerned with
  • a)
    explaining the effects that the burning of fossil fuels might have on climate
  • b)
    illustrating the effects of CO2 on atmospheric radiation
  • c)
    discussing effects that changes in the CO2 level in the atmosphere might have on climate
  • d)
    challenging hypotheses about the effects of water vapor and CO2 on climate
  • e)
    refuting hypotheses by climatologists about the causes of global temperature fluctuations
Correct answer is option 'C'. Can you explain this answer?

Advait Roy answered
Focus of the Author
The author's primary focus is on the implications of changes in the CO2 levels in the atmosphere and their subsequent effects on climate. This is evident through various discussions and theories presented in the text.
Key Points Supporting Option C
- Impact of CO2 Changes: The text emphasizes how variations in CO2 concentration can affect global temperatures. For instance, theories suggest that halving CO2 could lead to significant cooling or a moderate decrease in temperature.
- Oscillation of CO2 Levels: The narrative outlines a cyclical process where increased CO2 from natural events leads to warmer climates, promoting plant growth that eventually reduces CO2 levels and causes cooling. This illustrates a direct relationship between CO2 levels and climatic changes.
- Climate Feedback Mechanisms: The author discusses feedback loops involving CO2 and temperature, highlighting how changes in one can lead to fluctuations in the other, which is essential to understanding climate dynamics.
- Critique of Temperature Measurement: There is a critical view of global temperature assessments, suggesting that observed increases might not be uniform across the globe due to limitations in meteorological data, further supporting the complexity of CO2's role in climate change.
Conclusion
Overall, the author provides a comprehensive analysis of how alterations in atmospheric CO2 levels can significantly impact climate, underscoring the intricate relationship between these variables. This aligns with option C, as it delves deeply into the effects of changing CO2 levels on climate systems.

John, a stock broker, has a list of companies whose shares he recommends his clients to invest in. Over the past one year, the share price of 20 companies listed on the stock exchange has appreciated by 100% or more and 16 of these companies are part of John’s list. Thus John claims that he is an expert at picking stocks and that more and more investors should park their funds with him to get the maximum return on their investment.
The answer to which of the following questions would be most important in determining whether an investor should park his funds with John?
  • a)
    How many companies are there on John’s list?
  • b)
    Whether any other stock broker has also showed similar or better performance last year?
  • c)
    What is the total number of companies listed on the stock exchange?
  • d)
    Has John shown similar results in the previous years as well?
  • e)
    Does John hire the services of someone else to identify stocks in which his clients should invest?
Correct answer is option 'B'. Can you explain this answer?

Advait Malik answered
Understanding John's Claim
John claims to be an expert stock picker because 16 out of 20 companies he recommends have appreciated by 100% or more over the past year. However, to evaluate the validity of his claim, it's crucial to compare his performance with other brokers.
Importance of Comparison
- Performance Benchmark: Knowing whether other brokers have achieved similar or better results provides context to John's success. If many brokers have similar or superior performance, John's claims may not stand out, suggesting that his expertise might not be as exceptional as he presents.
- Market Trends: If the stock market as a whole experienced significant growth, it could indicate that many stocks appreciated, not necessarily due to John's expertise.
- Investor Decision Making: Investors need to assess whether John’s stock picks are genuinely better than those of competitors. A robust track record compared to others is essential for building trust.
Why Other Options Are Less Relevant
- Number of Companies on John's List: This does not directly indicate his expertise or the quality of his recommendations.
- Total Companies Listed on the Exchange: This figure does not provide insight into John's specific performance.
- Past Performance: While it is helpful, it doesn't address the current competitive landscape.
- Hiring Practices: Understanding if John hires others does not directly impact the assessment of his stock-picking ability.
In conclusion, evaluating John's performance against other brokers is crucial in determining whether investors should trust him with their funds.

Ramirez: The film industry claims that pirated DVDs, which are usually cheaper than legitimate DVDs and become available well before a film's official DVD release date, adversely affect its bottom line. But the industry should note what the spread of piracy indicates: consumers want lower prices and faster DVD releases. Lowering prices of DVDs and releasing them sooner would mitigate piracy's negative effect on film industry profits.

The argument above relies on which of the following assumptions?
  • a)
    Releasing legitimate DVDs earlier would not cause any reduction in the revenue the film industry receives from the films' theatrical release.
  • b)
    Some people who would otherwise purchase pirated DVDs would be willing to purchase legitimate DVDs if they were less expensive and released earlier than they are now.
  • c)
    The film industry will in the future be able to produce DVDs more cheaply than is currently the case.
  • d)
    Some current sellers of pirated DVDs would likely discontinue their businesses if legitimate DVDs were released faster and priced lower.
  • e)
    Current purchasers of pirated DVDs are aware that those DVDs are not authorized by the film industry.
Correct answer is option 'B'. Can you explain this answer?

Rahul Kapoor answered
B. Some people who would otherwise purchase pirated DVDs would be willing to purchase legitimate DVDs if they were less expensive and released earlier than they are now.
This assumption suggests that there is a potential market for legitimate DVDs among those who currently purchase pirated DVDs. If legitimate DVDs were more affordable and released earlier, these consumers would be willing to choose the legitimate option instead. This assumption forms the basis for the argument's claim that lowering prices and releasing DVDs sooner would mitigate the negative impact of piracy on film industry profits.

A research on early language development concluded that children born in English-speaking countries acquire better skills in English by the age of 5 than children born in non-English-speaking countries. Therefore, if a newly married couple from a non- English-speaking country were to begin their family in an English-speaking country and stay there until their children turn at least 5, their children are expected to acquire better skills in English than they would likely have acquired if the family had continued to stay in its home country.
Which one of the following, if true, would most seriously weaken the conclusion drawn in the passage?
  • a)
    Experts in early language development do not believe that children born in English-speaking countries acquire better skills in English by the age of 5 than children born in non-English-speaking countries.
  • b)
    The research mentioned was carried out with only 100 children and is, therefore, statistically unreliable.
  • c)
    The language spoken at home plays a crucial role in a child's early language development.
  • d)
    There are some parents from non-English-speaking countries who speak better English than some parents from English- speaking countries.
  • e)
    Staying in an English-speaking country gives better exposure to English than staying in a non-English-speaking country, and thus improves one's skills in English.
Correct answer is option 'C'. Can you explain this answer?

Rahul Kapoor answered
The argument in the passage states that children born in English-speaking countries acquire better English language skills by the age of 5 compared to children born in non-English-speaking countries. It further concludes that if a couple from a non-English-speaking country were to start their family in an English-speaking country and stay there until their children turn at least 5, their children are expected to acquire better English skills than if they had stayed in their home country.
We are asked to find an option that weakens this conclusion the most. Let's analyze each option:
(a) Experts in early language development do not believe that children born in English-speaking countries acquire better skills in English by the age of 5 than children born in non-English-speaking countries.
This option directly contradicts the premise of the argument and, if true, would weaken the conclusion. However, we are looking for the option that weakens the conclusion the most, and this option does not provide any additional information or explanation.
(b) The research mentioned was carried out with only 100 children and is, therefore, statistically unreliable.
This option attacks the reliability of the research mentioned in the passage. If the research is statistically unreliable, then the conclusion based on that research becomes less convincing. However, this option does not address the specific comparison between children born in English-speaking countries and those born in non-English-speaking countries.
(c) The language spoken at home plays a crucial role in a child's early language development.
This option weakens the conclusion by introducing another factor that may influence a child's language development: the language spoken at home. It suggests that even if the family moves to an English-speaking country, if they continue speaking their native language at home, it could impact the child's English language acquisition. This option provides an alternative explanation for the difference in language skills observed between the two groups.
(d) There are some parents from non-English-speaking countries who speak better English than some parents from English-speaking countries.
This option introduces variability within both groups, indicating that language skills are not solely determined by the country of birth. While it challenges the generalization made in the conclusion, it does not specifically weaken the comparison between the two groups of children.
(e. Staying in an English-speaking country gives better exposure to English than staying in a non-English-speaking country, and thus improves one's skills in English.
This option supports the conclusion by providing a reason why children in English-speaking countries might have better English language skills. It does not weaken the argument but rather reinforces it.
Therefore, option (c) weakens the conclusion the most by introducing the importance of the language spoken at home as a crucial factor in a child's early language development, suggesting that the country of residence alone may not be the sole determining factor in language acquisition.

Kazuko Nakane’s history of the early Japanese immigrants to central California’s Pajaro Valley focuses on the development of farming communities there from 1890 to 1940. The Issei (first-generation immigrants) were brought into the Pajaro Valley to raise sugar beets. Like Issei laborers in American cities, Japanese men in rural areas sought employment via the “boss” system. The system comprised three elements: immigrant wage laborers; Issei boardinghouses where laborers stayed; and labor contractors, who gathered workers for a particular job and then negotiated a contract between workers and employer. This same system was originally utilized by the Chinese laborers who had preceded the Japanese. A related institution was the “labor club,” which provided job information and negotiated employment contracts and other legal matters, such as the rental of land, for Issei who chose to belong and paid an annual fee to the cooperative for membership.

When the local sugar beet industry collapsed in 1902, the Issei began to lease land from the valley’s strawberry farmers. The Japanese provided the labor and the crop was divided between laborers and landowners. The Issei thus moved quickly from wage-labor employment to sharecropping agreements. A limited amount of economic progress was made as some Issei were able to rent or buy farmland directly, while others joined together to form farming corporations. As the Issei began to operate farms, they began to marry and start families, forming an established Japanese American community. Unfortunately, the Issei’s efforts to attain agricultural independence were hampered by government restrictions, such as the Alien Land Law of 1913. But immigrants could circumvent such exclusionary laws by leasing or purchasing land in their American-born children’s names.

Nakane’s case study of one rural Japanese American community provides valuable information about the lives and experiences of the Issei. It is, however, too particularistic. This limitation derives from Nakane’s methodology—that of oral history—which cannot substitute for a broader theoretical or comparative perspective. Future research might well consider two issues raised by her study: were the Issei of the Pajaro Valley similar to or different from Issei in urban settings, and what variations existed between rural Japanese American communities?
Several Issei families join together to purchase a strawberry field and the necessary farming equipment. Such a situation best exemplifies which of the following, as it is described in the passage?
  • a)
    A typical sharecropping agreement
  • b)
    A farming corporation
  • c)
    A “labor club”
  • d)
    The “boss” system
  • e)
    Circumvention of the Alien Land Law
Correct answer is option 'B'. Can you explain this answer?

EduRev GMAT answered
The situation where several Issei families join together to purchase a strawberry field and the necessary farming equipment best exemplifies:
(B) A farming corporation
Explanation:
In the passage, it is mentioned that some Issei were able to rent or buy farmland directly and that others joined together to form farming corporations. This describes a cooperative effort where multiple individuals or families pool their resources to collectively own and operate a farm. The described situation fits the definition of a farming corporation, where the combined efforts of Issei families result in shared ownership and operation of agricultural assets.
  • (A) is incorrect because a typical sharecropping agreement involves laborers working on someone else's land and splitting the crop with the landowners, which is different from purchasing and owning land.
  • (C) is incorrect because a “labor club” is an institution providing job information and negotiating contracts, not involved in purchasing farmland.
  • (D) is incorrect because the “boss” system is about employment and labor management, not about forming corporations or purchasing land.
  • (E) is incorrect because circumvention of the Alien Land Law involves leasing or purchasing land through American-born children, not forming farming corporations.
Therefore, (B) accurately describes the scenario of multiple Issei families joining together to buy and operate a farm.

Some philosophers explain visual perception by suggestion that when we visually perceive an object, a mental image of that object forms in our mind. However, this hypothesis cannot be correct, since it would require an inner self visually perceiving the newly formed mental image; this would in turn require that he inner self have a mental image of that mental image and so on. But such an infinite regress is absurd.

Which one of the following arguments is most similar in its pattern of reasoning to the argument above?
  • a)
    According to some linguists, many of the world's languages can be tracked back to a common source known as indo-European. However, Indo-European cannot be the earliest language for if it were, then there would be no language from which it was derived. But this is highly unlikely, given the overwhelming evidence that humans spoke long before the advent of Indo-European.
  • b)
    The claim that any scientific theory is adequate as long as it agrees with all the empirical data cannot be correct For there are an infinite number of theories all of which account equally well for the empirical data, and they cannot all be true at the same time.
  • c)
    Some historians claim that no theory is ever genuinely new; no matter how clever a theory is, there is always a precedent theory that contains its gist. But if this were true, then every theory would have a precedent theory contains its gist, and this precedent theory would have a precedent theory, and so on, without end. Since this is clearly impossible, the historians' claim must be false.
  • d)
    Some engineers define a structure's foundation as that part of the structure that supports the rest of the structure. This definition is unfortunate, however, because it evokes the suggestion that the foundation itself does not have any support, which, of course, is absurd.
  • e)
    Some people claim that the first library was the library of Alexandria, which for many centuries contained the largest collection of books in the world. However, Alexandria's collection was itself put together from smaller collections, small libraries in themselves. It follows that the library of Alexandria was not the first in the world.
Correct answer is option 'C'. Can you explain this answer?

Notes Wala answered
(A) According to some linguists, many of the world's languages can be tracked back to a common source known as Indo-European. However, Indo-European cannot be the earliest language for if it were, then there would be no language from which it was derived. But this is highly unlikely, given the overwhelming evidence that humans spoke long before the advent of Indo-European. This argument does not follow the same pattern of reasoning as the given argument. It presents a counter-argument based on the premise that if Indo-European were the earliest language, there would be no language from which it was derived. It does not involve an infinite regress.
(B) The claim that any scientific theory is adequate as long as it agrees with all the empirical data cannot be correct. For there are an infinite number of theories all of which account equally well for the empirical data, and they cannot all be true at the same time. This argument follows a similar pattern of reasoning. It claims that the given claim is incorrect because it would lead to a problem with an infinite number of theories, which cannot all be true at the same time. It involves an infinite regress-like problem, similar to the given argument.
(C) Some historians claim that no theory is ever genuinely new; no matter how clever a theory is, there is always a precedent theory that contains its gist. But if this were true, then every theory would have a precedent theory that contains its gist, and this precedent theory would have a precedent theory, and so on, without end. Since this is clearly impossible, the historians' claim must be false. This argument follows a similar pattern of reasoning. It argues against the historians' claim by pointing out that it would lead to an infinite regress, which is deemed impossible. It involves the same type of reasoning as the given argument.
(D) Some engineers define a structure's foundation as that part of the structure that supports the rest of the structure. This definition is unfortunate, however, because it evokes the suggestion that the foundation itself does not have any support, which, of course, is absurd. This argument does not follow the same pattern of reasoning. It presents a definition and claims that it evokes an absurd suggestion, without involving an infinite regress.
(E) Some people claim that the first library was the library of Alexandria, which for many centuries contained the largest collection of books in the world. However, Alexandria's collection was itself put together from smaller collections, small libraries in themselves. It follows that the library of Alexandria was not the first in the world. This argument does not follow the same pattern of reasoning. It presents evidence that challenges the claim about the first library by pointing out that Alexandria's collection was compiled from smaller collections. It does not involve an infinite regress.
Based on the evaluation, the argument that is most similar in its pattern of reasoning to the given argument is (C) Some historians claim that no theory is ever genuinely new; no matter how clever a theory is, there is always a precedent theory that contains its gist. But if this were true, then every theory would have a precedent theory that contains its gist, and this precedent theory would have a precedent theory, and so on, without end. Since this is clearly impossible, the historians' claim must be false.

Investing in fishing-boats could be very profitable at this time. A survey made by "Hook, Line and Sinker" magazine shows that 75 percent of the magazine's readers want to buy a new fishing-boat during the summer. However, fishing-boat manufacturers can only produce enough boats to satisfy 30% of total potential buyers.
Which of the following, if true, reveals a weakness in the evidence cited above?
  • a)
    The fishing-boat industry is a highly labor-intensive business.
  • b)
    Fishing-boats are not evenly distributed across the country.
  • c)
    The number of fishermen who buy fishing-boats has been growing each year for the past six years.
  • d)
    Readers of "Hook, Line and Sinker" magazine are more likely than other consumers to want a fishing-boat.
  • e)
    "Hook, Line and Sinker" magazine includes both articles about fishing and articles about building fishing-boats.
Correct answer is option 'D'. Can you explain this answer?

Rahul Kapoor answered
Information provided:
  • "Hook, Line and Sinker" magazine conducted a survey.
  • 75% of the magazine's readers want to buy a new fishing-boat during the summer.
  • Fishing-boat manufacturers can only produce enough boats to satisfy 30% of total potential buyers.
We are asked to identify a weakness in the evidence provided.
(A) The fishing-boat industry is a highly labor-intensive business.
This answer choice does not weaken the evidence presented. The labor intensity of the fishing-boat industry does not directly relate to the survey results or the boat manufacturers' production capacity.
(B) Fishing-boats are not evenly distributed across the country.
This answer choice does not directly weaken the evidence presented. Uneven distribution of fishing boats does not necessarily indicate a weakness in the survey results or the production capacity of manufacturers.
(C) The number of fishermen who buy fishing-boats has been growing each year for the past six years.
This answer choice does not weaken the evidence. The fact that the number of fishermen buying boats has been growing does not provide any information about the current situation or the percentage of potential buyers satisfied by the manufacturers.
(D) Readers of "Hook, Line and Sinker" magazine are more likely than other consumers to want a fishing-boat.
This answer choice weakens the evidence because it introduces a potential bias in the survey results. If readers of the magazine are more likely than other consumers to want a fishing boat, then the survey results might not accurately represent the general population. The survey's findings may not reflect the overall demand for fishing boats accurately.
(E) "Hook, Line and Sinker" magazine includes both articles about fishing and articles about building fishing-boats.
This answer choice does not weaken the evidence. The inclusion of articles about building fishing boats does not directly affect the survey results or the manufacturers' production capacity.
Therefore, the correct answer is (D). Readers of "Hook, Line and Sinker" magazine being more likely than other consumers to want a fishing-boat weakens the evidence provided by the survey results, as it introduces a potential bias.

Not all life depends on energy from sunlight. Microbial life has been found in bedrock more than five kilometers below the surface of the Earth, and bacteria have been found on the deep ocean floor feeding on hydrogen and other gases rising from the interior of the Earth through vents in the ocean floor.
The statements above, if true, best support which of the following as a conclusion?
  • a)
    The location in the bedrock where microbial life was found was not near a system of volcanic vents through which hydrogen and other gases rose from the interior of the Earth.
  • b)
    Bacteria are able to exist at the molten center of the Earth.
  • c)
    A thorough survey of a planet's surface is insufficient to establish beyond a doubt that the planet contains no life.
  • d)
    Life probably exists on Sun-orbiting comets, which are cold agglomerations of space dust and frozen gases.
  • e)
    Finding bacterial remains in coal and oil would establish that the bacteria had been feeding on substances that had not been produced from the energy of sunlight.
Correct answer is option 'C'. Can you explain this answer?

Rahul Kapoor answered
(C) A thorough survey of a planet's surface is insufficient to establish beyond a doubt that the planet contains no life.
The statements highlight examples of microbial life thriving in extreme environments on Earth, such as deep within the bedrock and on the ocean floor. These findings indicate that life can exist in unexpected and challenging conditions. Therefore, the conclusion that a thorough survey of a planet's surface is insufficient to definitively establish the absence of life is supported. The presence of microbial life in unconventional locations on Earth suggests that there may be undiscovered forms of life elsewhere, even in environments that may appear inhospitable at first glance.

People waste a surprising amount of money on gadgets and doodads that they hardly ever use. For example, my brother spent $25 on an electric ice-cream maker two years ago, but he has used it on only three occasions. Yet, he insists that regardless of the number of times he actually uses the ice-cream maker, the investment was a good one because ___________.
Which of the following best completes the thought of the paragraph?
  • a)
    the price of ice cream will go up in the future
  • b)
    he has purchased the ice-cream maker for the convenience of having it available if and when he needs it
  • c)
    in a society that is oriented toward consumer goods, one should take every opportunity to acquire things
  • d)
    today $25 is not worth what it was two years ago on account of the inflation rate
  • e)
    by using it so infrequently he has conserved a considerable amount of electrical energy
Correct answer is option 'B'. Can you explain this answer?

Rahul Kapoor answered
A. The price of ice cream will go up in the future: This option suggests that the investment in the ice-cream maker was a good one because the price of ice cream is expected to increase in the future. However, this reasoning is not explicitly stated or implied in the paragraph, so it does not logically complete the thought.
B. He has purchased the ice-cream maker for the convenience of having it available if and when he needs it: This option aligns with the idea of justifying the investment in the ice-cream maker despite its infrequent use. It suggests that the value of the purchase lies in the convenience of having the ice-cream maker accessible for future occasions when it may be needed. This option completes the thought in a logical and relevant manner.
C. In a society that is oriented toward consumer goods, one should take every opportunity to acquire things: This option presents a broader perspective on consumer behavior and suggests that acquiring things is important in a society focused on consumer goods. However, this idea does not directly address or support the reasoning behind the brother's justification for the investment in the ice-cream maker.
D. Today $25 is not worth what it was two years ago on account of the inflation rate: This option introduces the concept of inflation and suggests that the value of $25 has decreased over time. However, this reasoning does not directly relate to the brother's justification for the investment in the ice-cream maker.
E. By using it so infrequently, he has conserved a considerable amount of electrical energy: This option focuses on the energy-saving aspect of infrequently using the ice-cream maker. While it highlights a positive outcome of the brother's behavior, it does not directly address or support his justification for the investment.
Among the options provided, option B is the best completion because it aligns with the brother's perspective on the value of the ice-cream maker based on convenience and availability.

The city of Workney, in raising bus fares from $1.00 to $1.25, proposed that 18 fare token be sold for $20.00 to alleviate the extra burden of the fare increase on the city's low-income residents. Critics suggested alternatively that 9 fare tokens be sold for $10.00, because a $20.00 outlay would be prohibitive for low-income riders.
The alternative proposal depends on which of the following assumptions?
  • a)
    Low-income residents of Workney will continue to ride the buses in the same numbers despite the fare increase
  • b)
    Low-income riders would be more likely to take advantage of the savings afforded by the 9-token offer than would other riders.
  • c)
    The outlay of $10.00 for the purchase of 9 fare tokens would not be prohibitive for low-income bus riders
  • d)
    The proposed fare increase is needed for the purchase of new buses for the city's bus system
  • e)
    Buses provide the only means of public transportation in the city of Workney.
Correct answer is option 'C'. Can you explain this answer?

A. Low-income residents of Workney will continue to ride the buses in the same numbers despite the fare increase.
This assumption suggests that the fare increase will not significantly impact the ridership of low-income residents. If they continue to ride the buses in the same numbers, it supports the feasibility of the alternative proposal.
B. Low-income riders would be more likely to take advantage of the savings afforded by the 9-token offer than would other riders.
This assumption implies that low-income riders are more price-sensitive and would be more inclined to take advantage of the cost-saving opportunity provided by the 9-token offer. It suggests that the alternative proposal would be beneficial specifically for low-income riders.
C. The outlay of $10.00 for the purchase of 9 fare tokens would not be prohibitive for low-income bus riders.
This assumption is crucial for the alternative proposal to be a viable option for low-income riders. It suggests that the cost of purchasing 9 fare tokens for $10.00 would not impose a significant financial burden on low-income bus riders.
D. The proposed fare increase is needed for the purchase of new buses for the city's bus system.
This assumption suggests that the fare increase is necessary to generate additional revenue for the city's bus system, potentially to fund the purchase of new buses. It implies that the fare increase is justified and serves a specific purpose.
E. Buses provide the only means of public transportation in the city of Workney.
This assumption implies that the bus system is the primary mode of public transportation in Workney and that alternative transportation options are limited or nonexistent. It underscores the importance of addressing the fare increase issue for low-income residents who heavily rely on buses for their transportation needs.
It's important to note that while all of these assumptions play a role in evaluating the alternative proposal, the strongest assumption that directly supports the feasibility of the alternative proposal is option C.

A study of marital relationships in which one partner's sleeping and waking cycles differ from those of the other partner reveals that such couples share fewer activities with each other and have more violent arguments than do couples in a relationship in which both partners follow the same sleeping and waking patterns. Thus, mismatched sleeping and waking cycles can seriously jeopardize a marriage.

Which of the following, if true, most seriously weakens the argument above?
  • a)
    Married couples in which both spouses follow the same sleeping and waking patterns also occasionally have arguments that can jeopardize the couple's marriage.
  • b)
    The sleeping and waking cycles of individuals tend to vary from season to season.
  • c)
    The individuals who have sleeping and waking cycles that differ significantly from those of their spouses tend to argue little with colleagues at work.
  • d)
    People in unhappy marriages have been found to express hostility by adopting a different sleeping and waking cycle from that of their spouses.
  • e)
    According to a recent study, most people's sleeping and waking cycles can be controlled and modified easily.
Correct answer is option 'D'. Can you explain this answer?

Rahul Kapoor answered
(A) Married couples in which both spouses follow the same sleeping and waking patterns also occasionally have arguments that can jeopardize the couple's marriage. This option does not weaken the argument. The fact that couples with matching sleeping and waking patterns also have arguments does not address the impact of mismatched cycles on marital relationships.
(B) The sleeping and waking cycles of individuals tend to vary from season to season. This option does not weaken the argument. Although individuals' sleeping and waking cycles may vary from season to season, it does not directly challenge the negative effects of mismatched cycles on marital relationships.
(C) The individuals who have sleeping and waking cycles that differ significantly from those of their spouses tend to argue little with colleagues at work. This option does not weaken the argument. The argument focuses on the impact of mismatched cycles on marital relationships, not the relationships with colleagues at work.
(D) People in unhappy marriages have been found to express hostility by adopting a different sleeping and waking cycle from that of their spouses. This option weakens the argument. It suggests that mismatched sleeping and waking cycles could be a result of unhappiness in marriages, rather than the cause of the negative effects on marital relationships.
(E) According to a recent study, most people's sleeping and waking cycles can be controlled and modified easily. This option does not weaken the argument. Even if people's sleeping and waking cycles can be controlled and modified easily, it does not address the impact of mismatched cycles on marital relationships.
Based on the analysis, option (D) weakens the argument by providing an alternative explanation for the observed negative effects on marital relationships.

Manager: the new manufacturing process should save us time overall, even though the first step of the five-step process will take twice as long as it does under the old process. Under the new process, far fewer of the components will be found defective, and the sole purpose of steps two and three under the old process is to weed out defective components. As a result, we should be able to eliminate two of the five steps in the existing manufacturing process.
Which of the following would be most useful in evaluating the claim made in the argument?
  • a)
    Whether factory workers will require training in order to use the new manufacturing process
  • b)
    Whether the new process is likely to introduce deficiencies or imperfections that must be corrected
  • c)
    Whether defective components can be fixed or must be thrown out
  • d)
    Whether a third manufacturing process would save even more time than both the old and new manufacturing processes
  • e)
    Whether saving time with the new manufacturing process will ultimately lead to cost savings for the company.
Correct answer is option 'B'. Can you explain this answer?

Mihir Ghoshal answered
Evaluation of the Argument
The manager's claim presents a new manufacturing process that purportedly saves time despite a longer initial step. To evaluate this claim effectively, it's essential to consider potential drawbacks of the new process.
Importance of Assessing Deficiencies
- Deficiencies and Imperfections: The focus on whether the new process might introduce deficiencies or imperfections is crucial. If the new process results in new defects, it could negate the time savings achieved in the first step.
- Quality Assurance: The argument hinges on the assumption that the new process will yield fewer defective components. If the new process introduces its own set of quality issues, the elimination of steps two and three may not be justified.
Implications of Defects
- Impact on Overall Efficiency: If defects arise from the new process, it may require additional time and resources for corrections, thereby undermining the claimed time savings.
- Long-Term Viability: Understanding whether the new process is sustainable in terms of quality will determine if the time savings translate into a reliable manufacturing method.
Conclusion
By investigating whether the new manufacturing process is likely to introduce deficiencies or imperfections, stakeholders can make a more informed decision about its overall effectiveness and whether the time savings are truly beneficial. Thus, option 'B' serves as the most useful avenue for evaluating the manager's claim.

In a recent study, Mario García argues that in the United States between 1930 and 1960 the group of political activists he calls the “Mexican American Generation” was more radical and politically diverse (5) than earlier historians have recognized. Through analysis of the work of some of the era’s most important scholars, García does provide persuasive evidence that in the 1930s and 1940s these activists anticipated many of the reforms proposed by the more (10) militant Chicanos of the 1960s and 1970s. His study, however, suffers from two flaws.
First, García’s analysis of the evidence he provides to demonstrate the Mexican American Generation’s political diversity is not entirely (15) consistent. Indeed, he undermines his primary thesis by emphasizing an underlying consensus among various groups that tends to conceal the full significance of their differences. Groups such as the League of United Latin American Citizens, an (20) organization that encouraged Mexican Americans to pursue a civil rights strategy of assimilation into the United States political and cultural mainstream, were often diametrically opposed to organizations such as the Congress of Spanish-Speaking People, a coalition (25) group that advocated bilingual education and equal rights for resident aliens in the United States. García acknowledges these differences but dismisses them as insignificant, given that the goals of groups as disparate as these centered on liberal reform, not (30) revolution. But one need only note the fierce controversies that occurred during the period over United States immigration policies and the question of assimilation versus cultural maintenance to recognize that Mexican American political history since 1930 (35) has been characterized not by consensus but by intense and lively debate.
Second, García may be exaggerating the degree to which the views of these activists were representative of the ethnic Mexican population residing in the (40) United States during this period. Noting that by 1930 the proportion of the Mexican American population that had been born in the United States had significantly increased, García argues that between 1930 and 1960 a new generation of Mexican American (45) leaders appeared, one that was more acculturated and hence more politically active than its predecessor. Influenced by their experience of discrimination and by the inclusive rhetoric of World War II slogans, these leaders, according to García, were determined to (50) achieve full civil rights for all United States residents of Mexican descent. However, it is not clear how far this outlook extended beyond these activists. Without a better understanding of the political implications of important variables such as patterns of bilingualism (55) and rates of Mexican immigration and naturalization, and the variations in ethnic consciousness these variables help to create, one cannot assume that an increase in the proportion of Mexican Americans born in the United States necessarily resulted in an increase (60) in the ethnic Mexican population’s political activism.
The passage suggests that García assumes which one of the following to have been true of Mexican Americans between 1930 and 1960?
  • a)
    Increased ethnic consciousness among Mexican Americans accounted for an increase in political activity among them.
  • b)
    Increased familiarity among Mexican Americans with United States culture accounted for an increase in political activity among them.
  • c)
    The assimilation of many Mexican Americans into United States culture accounted for Mexican Americans’ lack of interest in political activity.
  • d)
    Many Mexican Americans were moved to political militancy as a means of achieving full civil rights for all United States residents of Mexican descent.
  • e)
    Many Mexican Americans were moved to political protest by their experience of discrimination and the patronizing rhetoric of World War II slogans.
Correct answer is option 'B'. Can you explain this answer?

Wizius Careers answered
To determine which statement the passage suggests García assumes to be true, let’s examine each option in the context of the passage:
  1. Increased Ethnic Consciousness and Political Activity: The passage questions the assumption that an increase in the proportion of Mexican Americans born in the U.S. necessarily led to increased political activism. García argues that these new leaders were more acculturated and politically active, but the passage challenges this link by highlighting the need for a better understanding of factors like bilingualism and immigration patterns. This option does not align with García’s assumption but rather with the passage’s critique of it.
  2. Increased Familiarity with U.S. Culture and Political Activity: García argues that between 1930 and 1960, a more acculturated generation of Mexican American leaders emerged. García seems to assume that increased familiarity with U.S. culture, resulting from the higher proportion of U.S.-born Mexican Americans, contributed to increased political activity. This assumption is suggested by García’s argument that these leaders were more politically active due to their experience and acculturation.
  3. Assimilation and Lack of Political Activity: The passage does not suggest García believes that assimilation led to a lack of interest in political activity. Instead, García's work argues that the more acculturated leaders were more politically active, so this option is inconsistent with García’s assumptions.
  4. Political Militancy for Full Civil Rights: While García discusses how the new generation of leaders was determined to achieve full civil rights, the passage does not specifically state that García assumes this determination was the primary motivation for political militancy.
  5. Political Protest from Discrimination and World War II Rhetoric: García notes that the new leaders were influenced by discrimination and World War II rhetoric. However, this influence does not directly address García’s assumption about increased familiarity with U.S. culture and its impact on political activity.
Correct Option B:
Explanation: The passage implies that García assumes the increased familiarity of Mexican Americans with U.S. culture (through increased proportion of U.S.-born individuals) contributed to their heightened political activity. The passage critiques this assumption but does not dispute that García made it. Hence, option B is the best fit.

A certain credit-card company awards its customers bonus points for using its credit card. Customers can use accumulated points in the purchase of brand name merchandise by mail at prices lower than the manufacturers‘ suggested retail prices. At any given time, therefore, customers who purchase merchandise using the bonus points spend less than they would spend if they purchased the same merchandise in retail stores.
Which one of the following is an assumption on which the argument depends?
  • a)
    The merchandise that can be ordered by mail using the bonus points is not oflcrcd at lower prices by other credit-card companies that award bonus points.
  • b)
    The bonus points cannot be used by the credit-card customers in the purchase of brand name merchandise that is not available for purchase in retail stones.
  • c)
    The credit-card company does not require its customers to accumulate a large number of bonus points before becoming eligible to order merchandise at prices lower than the manufacturers‘ suggested retail price.
  • d)
    The amount credit-card customers pay for shipping the merchandise ordered by mail does not increase the amount customers spend to an amount greater than they would spend if they purchased the same merchandise in retail stores.
  • e)
    The merchandise available to the company's credit-card customers using the bonus points is frequently sold in retail stores at prices that are higher than the manufacturers‘ suggested retail prices?
Correct answer is option 'D'. Can you explain this answer?

Rahul Kapoor answered
The argument states that customers of a certain credit-card company can use bonus points to purchase brand name merchandise at prices lower than the manufacturers' suggested retail prices. Based on this information, the argument concludes that customers spending their bonus points will spend less than if they purchased the same merchandise in retail stores.
To weaken or challenge this argument, we need to identify an assumption on which the argument depends. In other words, we need to find an answer choice that, if false, would undermine the conclusion of the argument.
Let's examine each answer choice:
(A) The merchandise that can be ordered by mail using the bonus points is not offered at lower prices by other credit-card companies that award bonus points.
This answer choice is not relevant to the argument since it talks about other credit-card companies. The argument only concerns the specific credit-card company mentioned. So, it is not an assumption on which the argument depends.
(B) The bonus points cannot be used by the credit-card customers in the purchase of brand name merchandise that is not available for purchase in retail stores.
This answer choice is also not relevant to the argument. It talks about merchandise that is not available in retail stores, which is outside the scope of the argument. Therefore, it is not an assumption on which the argument depends.
(C) The credit-card company does not require its customers to accumulate a large number of bonus points before becoming eligible to order merchandise at prices lower than the manufacturers' suggested retail price.
This answer choice does not impact the conclusion of the argument. Even if customers need to accumulate a large number of bonus points, it does not affect the claim that they will spend less when using those points compared to retail store prices. Hence, it is not an assumption on which the argument depends.
(D) The amount credit-card customers pay for shipping the merchandise ordered by mail does not increase the amount customers spend to an amount greater than they would spend if they purchased the same merchandise in retail stores.
This answer choice directly addresses the comparison between the amount customers spend using bonus points and the amount they would spend in retail stores. If the shipping cost were to increase the total amount spent on the merchandise ordered by mail, it would weaken the argument's claim that customers spend less. Therefore, this is a crucial assumption on which the argument depends.
(E) The merchandise available to the company's credit-card customers using the bonus points is frequently sold in retail stores at prices that are higher than the manufacturers' suggested retail prices.
This answer choice is not relevant to the argument either. It talks about prices in retail stores, but the argument is specifically comparing the prices customers pay using bonus points with the manufacturers' suggested retail prices. So, it is not an assumption on which the argument depends.
Based on the analysis above, we can conclude that answer choice (D) is the assumption on which the argument depends. If the shipping cost were to increase the total amount spent on the merchandise ordered by mail, it would undermine the claim that customers spend less by using their bonus points.
Therefore, the correct answer is (D).

Politician: Most of those at the meeting were not persuaded by Kuyler’s argument, nor should they have been, for Kuyler’s argument implied that it would be improper to enter into a contract with the government; and yet—as many people know— Kuyler’s company has had numerous lucrative contracts with the government.
Which one of the following describes a flaw in the politician’s argument?
  • a)
    It concludes that an argument is defective merely on the grounds that the argument has failed to persuade anyone of the truth of its conclusion.
  • b)
    It relies on testimony that is likely to be biased.
  • c)
    It rejects an argument merely on the grounds that the arguer has not behaved in a way that is consistent with the argument.
  • d)
    It rejects a position merely on the grounds that an inadequate argument has been given for it.
  • e)
    It rejects an argument on the basis of an appeal to popular opinion.
Correct answer is option 'C'. Can you explain this answer?

(C) It rejects an argument merely on the grounds that the arguer has not behaved in a way that is consistent with the argument.
In the argument, the politician dismisses Kuyler's argument because Kuyler's company has had numerous lucrative contracts with the government, which contradicts the implication that entering into a contract with the government would be improper. However, this is a flawed approach because it rejects Kuyler's argument solely based on Kuyler's behavior or past actions, rather than addressing the merits of the argument itself. The fact that Kuyler's company has had contracts with the government does not automatically render Kuyler's argument invalid or irrelevant. The argument should be evaluated based on its own reasoning and evidence, rather than being dismissed solely due to the perceived inconsistency with the arguer's behavior.

The First Amendment prevents the government from violating a person's right to free speech. But it does not protect government employees from disciplinary action for all statements they make in the course of their official duties. However, laws should protect them in cases in which employees bring to light information that advances the public interest in honest government and the rule of law.

Which of the following would provide the best example for the argument above?
  • a)
    An employee of the Central Intelligence Agency reveals classified information to a foreign government.
  • b)
    A movie star loses popularity because a studio employee tells a magazine about the star's troubled personal life.
  • c)
    An energy company’s stock plunges after an employee publicizes its accounting records.
  • d)
    A prominent plastic surgeon loses her license after an employee sues her for malpractice after she operates on him.
  • e)
    A Congressional aide reveals cases of bribery in the House of Representatives.
Correct answer is option 'E'. Can you explain this answer?

A) An employee of the Central Intelligence Agency reveals classified information to a foreign government. This example does not align with the argument because revealing classified information to a foreign government is not an act that advances the public interest in honest government and the rule of law. It involves unauthorized disclosure of sensitive information.
B) A movie star loses popularity because a studio employee tells a magazine about the star's troubled personal life. This example is unrelated to government employees and their role in advancing the public interest in honest government and the rule of law. It does not align with the argument.
C) An energy company’s stock plunges after an employee publicizes its accounting records. This example does not involve government employees or their role in advancing the public interest in honest government and the rule of law. It is unrelated to the argument.
D) A prominent plastic surgeon loses her license after an employee sues her for malpractice after she operates on him. This example does not involve government employees or the public interest in honest government and the rule of law. It is unrelated to the argument.
E) A Congressional aide reveals cases of bribery in the House of Representatives. This example aligns with the argument as it involves a government employee (a Congressional aide) bringing to light cases of bribery in the House of Representatives. It demonstrates the importance of protecting employees who disclose information that advances the public interest in honest government and the rule of law.
Therefore, the best example for the argument above is (E) A Congressional aide reveals cases of bribery in the House of Representatives.

Read the passage and answer the question given below.
Is it possible to decrease inflation without causing a recession and its concomitant increase in unemployment? The orthodox answer is “no.” whether they support the “inertia” theory of inflation (that today’s inflation rate is caused by yesterday’s inflation, the state of the economic cycle, and external influences such as import prices) or the “rational expectations” theory (that inflation is caused by workers’ and employers’ expectations, coupled with a lack of credible monetary and fiscal policies), most economists agree that tight monetary and fiscal policies, which cause recessions, are necessary to decelerate inflation. They point out that in the 1980’s, many European countries and the United States conquered high (by these countries’ standards) inflation, but only by applying tight monetary and fiscal policies that sharply increased unemployment. Nevertheless, some governments’ policymakers insist that direct controls on wages and prices, without tight monetary and fiscal policies, can succeed in decreasing inflation. Unfortunately, because this approach fails to deal with the underlying causes of inflation, wage and price controls eventually collapse, the hitherto-repressed inflation resurfaces, and in the meantime, though the policymakers succeed in avoiding a recession, a frozen structure of relative prices imposes distortions that do damage to the economy’s prospects for long-term growth.
Q. Which of the following, if true, would most strengthen the author’s conclusion about the use of wage and price controls?
  • a)
    Countries that repeatedly use wage and price controls tend to have lower long-term economic growth rates than do other countries.
  • b)
    Countries that have extremely high inflation frequently place very stringent controls on wages and prices in an attempt to decrease the inflation.
  • c)
    Some countries have found that the use of wage and price controls succeeds in decreasing inflation but also causes a recession.
  • d)
    Policymakers who advocate the use of wage and price controls believe that these controls will deal with the underlying causes of inflation.
  • e)
    Policymakers who advocate the use of wage and price controls are usually more concerned about long-term economic goals than about short-term economic goals.
Correct answer is option 'A'. Can you explain this answer?

Rahul Kapoor answered
The author's conclusion is that wage and price controls, although they may avoid a recession in the short term, eventually lead to distortions and damage to the economy's long-term growth prospects. Option (A) states that countries that repeatedly use wage and price controls tend to have lower long-term economic growth rates than other countries. This information directly supports the author's argument by highlighting the negative impact of wage and price controls on long-term economic growth.

Behind every book review there are two key figures: a book review editor and a reviewer. Editors decide whether a book is reviewed in their publication, when the review appears, how long it is, and who writes the review.
When many periodicals feature the same books, this does not prove that the editors of different periodicals have not made individual decisions. Before publication, editors receive news releases and printer's proofs of certain books, signifying that the publishers will make special efforts to promote these books. They will be heavily advertised and probably be among the books that most bookstores order in quantity. Not having such books reviewed might give the impression that the editor was caught napping, whereas too many reviews of books that readers will have trouble finding in stores would be inappropriate. Editors can risk having a few of the less popular titles reviewed, but they must consider what will be newsworthy, advertised, and written about elsewhere.
If these were the only factors influencing editors, few books that stand little chance of selling well would ever be reviewed. But editors feel some concern about what might endure, and therefore listen to literary experts. A generation ago, a newspaper used a brilliant system of choosing which books to feature. The book review editor sent out a greater number of books than reviews he actually intended to publish. If a review was unenthusiastic, he reasoned that the book was not important enough to be discussed immediately, and if good reviews of enough other books came in, the unenthusiastic review might never be printed. The unenthusiastic reviewers were paid promptly anyway, but they learned that if they wanted their material to be printed, it was advisable to be kind.
Most editors print favorable and unfavorable reviews; however, the content of the review may be influenced by the editor. Some editors would actually feel that they had failed in their responsibility if they gave books by authors they admired to hostile critics or books by authors they disapproved of to critics who might favor them. Editors usually can predict who would review a book enthusiastically and who would tear it to shreds.
The main idea of the second paragraph is that
  • a)
    decisions made by book review editors are influenced by the business of selling books
  • b)
    book review editors must be familiar with all aspects of the book trade
  • c)
    advertising is the most important factor influencing book sales
  • d)
    book reviews usually have no influence on what books are ordered in quantity by stores
  • e)
    publishers deliberately try to influence the decisions of book review editors
Correct answer is option 'A'. Can you explain this answer?

Tejas Gupta answered
Overview of the Role of Book Review Editors
The second paragraph emphasizes the intricate relationship between book review editors and the commercial aspects of publishing, highlighting how these editors navigate the landscape of book promotion and sales.
Influence of Selling Books
- Book review editors make decisions based on the potential marketability of a book.
- They consider the visibility of titles that are heavily advertised and promoted by publishers.
- Reviews of popular books are prioritized to avoid the perception that editors are neglecting important titles.
Balancing Act
- Editors must balance reviewing upcoming bestsellers while also being open to less popular titles.
- There is an understanding that not all books will achieve commercial success, yet there’s a desire to promote literary merit.
Editorial Judgment
- Editors selectively choose reviews to create a narrative that aligns with what is deemed newsworthy and relevant to readers.
- They rely on literary experts and previous reviews to guide their selections and maintain a standard of quality.
Conclusion
The main idea reflects that editorial decisions are indeed influenced by the commercial landscape, as editors strive to align their reviews with what is likely to resonate with readers and the market, making option 'A' the correct answer. Editors aim to foster a publication that effectively promotes books within the context of their anticipated sales and cultural significance.

The city of Northtown collects an average of $2.2 million in business taxes per year. Neighboring Southtown collects an average of $1.8 million in business taxes per year. Both cities assess business taxes on net profits. In an attempt to attract new businesses to Southtown, the spokesperson for the chamber of commerce of that city uses these statistics to claim that Southtown’s lower business tax rate offers amore favorable environment for business than can be found in Northtown.

Which of the following, if true, would most seriously undermine the spokesperson’s argument?
  • a)
    Most tax revenue collected in Northtown comes from business taxes.
  • b)
    Most tax revenue collected in Southtown comes from business taxes.
  • c)
    The net profits generated by Northtown businesses are currently twice those of Southtown businesses.
  • d)
    Northtown has twice the population of Southtown.
  • e)
    Southtown businesses generate twice as much sales revenue as do Northtown businesses.
Correct answer is option 'C'. Can you explain this answer?

Notes Wala answered
(A) Most tax revenue collected in Northtown comes from business taxes.
This option does not directly undermine the spokesperson's argument. It simply indicates that business taxes are a significant source of revenue for Northtown. It does not provide any information about the relative tax rates or the business environment in Northtown and Southtown.
(B) Most tax revenue collected in Southtown comes from business taxes.
Similar to option (A), this option does not directly undermine the spokesperson's argument. It suggests that business taxes are a significant source of revenue for Southtown, but it does not provide information about the relative tax rates or the business environment in the two cities.
(C) The net profits generated by Northtown businesses are currently twice those of Southtown businesses.
This option seriously undermines the spokesperson's argument. Despite Southtown's lower business tax rate, if the net profits generated by businesses in Northtown are significantly higher than those in Southtown, it suggests that Northtown may offer a more favorable business environment overall. Higher net profits may compensate for the higher tax rate in Northtown, making the spokesperson's claim less convincing.
(D) Northtown has twice the population of Southtown.
This option does not directly undermine the spokesperson's argument. It provides information about the population size of the two cities, but it does not offer any insights into the tax rates or the business environment. The size of the population alone does not determine the attractiveness of a business environment.
(E) Southtown businesses generate twice as much sales revenue as do Northtown businesses.
This option does not directly undermine the spokesperson's argument. It provides information about the sales revenue generated by businesses in Southtown and Northtown, but it does not provide any information about the tax rates or the profitability of the businesses. Sales revenue alone does not determine the overall business environment or the impact of tax rates on businesses.

A basic principle of ecology is that population size is partly a reflection of available food resources. Recent experiments suggest that the relationship is more complex than formerly thought. Specifically, the browsing of certain rodents appears to trigger chemical reactions in food plants which, in turn, affect the size of the rodent populations. Two examples of such regulation have been reported.
Berger has demonstrated the power of a naturally occurring chemical called 6-MBOA to stimulate reproductive behavior in the mountain vole, a small mouse-like rodent. 6 MBOA forms in young grass in response to browsing by voles. Berger experimented by feeding oats coated with 6-MBOA to non-breeding winter populations of voles. After three weeks, she found a high incidence of pregnancy among females. Since the timing of reproduction is crucial to the short-lived vole in an environment in which the onset of vegetative growth may be considerably delayed, the phytochemical triggering of reproductive behavior represents a significant biological adaptation.
In an example reported by Bryant, plants appear to have developed a phytochemical defense against the depredations of snowshoe hares in Canada. Every ten years, for reasons that are unclear, the hare population swells. The result is overbrowsing of certain deciduous trees and shrubs. Bryant found that trees favored by the hare produce young shoots high in terpene and phenolic resins, which discourage hare browsing. After treating non-resinous willow twigs with resinous extracts and placing treated and untreated samples at hare feeding stations, Bryant found that samples containing at least half of the resin concentration of natural twigs were untouched. The avoidance of resinous shoots, he concludes, may play a role in the decline of the hare population to normal levels.
Both of these reports suggest areas for further research. For example, data should be reviewed to determine if periodic population explosions among lemmings (another small rodent living in a northern environment) occur during years in which there is an early onset of vegetative growth; if so, a triggering mechanism similar to that prompted by the vole may be involved.
The passage describes the effect of 6-MBOA on voles as a “significant biological adaptation” because it:
  • a)
    limits reproductive behavior in times of food scarcity
  • b)
    prompts the vole population to seek new food sources
  • c)
    supports species survival during periods of fluctuating food supply
  • d)
    maximizes the number of offspring in individual litters
  • e)
    minimizes territorial competition
Correct answer is option 'C'. Can you explain this answer?

EduRev GMAT answered
The passage states that the formation of 6-MBOA in young grass in response to vole browsing triggers reproductive behavior in voles. This adaptation is significant because it allows the voles to time their reproduction with the availability of food resources, thus supporting their survival during periods when food supply might fluctuate.
Option A: The passage does not indicate that 6-MBOA limits reproductive behavior in times of food scarcity.
Option B: There's no information about 6-MBOA prompting the voles to seek new food sources.
Option D: The passage does not mention the effect of 6-MBOA on the number of offspring in individual litters.
Option E: The passage does not discuss territorial competition among voles in relation to 6-MBOA.

Most research on the subject has assumed that users of new technologies learn about and modify new technologies gradually. This assumption underlies the concept of the learning curve, a familiar model which presumes that learners gain knowledge in a continuous and highly regular progression over time.
However, when a new technology is introduced into a particular sector of a company, the process of learning about and modifying that technology may not, in fact, be continuous. In general, the introduction of new technology into ongoing operations triggers an initial burst of learning or adaptive activity as users explore the technology and attempt to resolve unexpected problems. Such intense activity is often short-lived, however, with users' effort and attention declining dramatically after the first few months. This rapid decline tends to occur even when serious problems remain unresolved, a trend which suggests that the dramatic falloff in learning or adaptive activity shortly after new technology is introduced does not reflect users' having mastered the new technology. The period of intense activity by users is typically followed by one in which users return their attention to their customary production tasks, though using the new technology to accomplish those tasks. Later on, particularly when challenges related to the new technology arise, users often turn their attention back to the new technology, initiating additional spurts of learning or adaptive activity. In many cases, this pattern continues over time, with brief periods of learning or adaptation followed by longer periods of relatively routine use.
Understanding that the pattern of user learning and adaptation is discontinuous can yield important benefits. Managers who anticipate and plan for intense user activity when a new technology is first introduced are better able to exploit the surge of energy and user motivation that usually occurs at the start of a project when improvements are easiest to implement, and major problems are most obvious. Moreover, by attempting to confine major modifications to start- up periods, such managers can better realize the benefits of periods of routine usage of the technology, during which productivity typically begins to increase and the effectiveness of previous modifications can be assessed.
According to information contained in the passage, the author would be most likely to make which of the following recommendations to managers who are introducing a new technology into the workplace?
  • a)
    Emphasize the desirability of maintaining certain productivity levels during all phases of the process.
  • b)
    Exploit the new technology by anticipating and planning for the continuous learning curve that characterizes this process.
  • c)
    Ensure that needed improvements to the new technology are implemented during the initial period of intense user learning and adaptive activity.
  • d)
    Work jointly with users to determine the amount of learning and adaptive activity that is required for users to master a new technology.
  • e)
    Attempt to minimize the dramatic falloff in learning and adaptive activity that often occurs shortly after a new technology is introduced.
Correct answer is option 'C'. Can you explain this answer?

EduRev GMAT answered
(A) suggests that managers should prioritize maintaining productivity throughout the entire process of introducing a new technology. However, the passage doesn't explicitly emphasize maintaining productivity as the primary concern. Instead, it highlights the cyclical nature of user learning and adaptation, with periods of intense activity followed by routine use.
(B) This option recommends planning for a continuous learning curve. However, the passage argues against this assumption by explaining that learning and adaptation to new technology are not continuous but rather have periods of intense activity followed by routine use.
(C) This option aligns with the passage. It suggests that managers should take advantage of the initial surge of user activity and motivation to make necessary improvements to the technology. The passage implies that this is a beneficial approach.
(D) This option suggests a collaborative approach between managers and users to determine the learning needs. The passage doesn't explicitly address this recommendation. It focuses more on the pattern of user activity and how managers can harness it.
(E) This is also incorrect.

Educator: Only those students who are genuinely curious about a topic can successfully learn about that topic. They find the satisfaction of their curiosity intrinsically gratifying, and appreciate the inherent rewards of the learning process itself. However, almost no child enters the classroom with sufficient curiosity to learn successfully all that the teacher must instill. A teacher’s job, therefore, _______.
Which one of the following most logically completes the educator’s argument?
  • a)
    requires for the fulfillment of its goals the stimulation as well as the satisfaction of curiosity
  • b)
    necessitates the creative use of rewards that are not inherent in the learning process itself
  • c)
    is to focus primarily on those topics that do not initially interest the students
  • d)
    is facilitated by students’ taking responsibility for their own learning
  • e)
    becomes easier if students realize that some learning is not necessarily enjoyable
Correct answer is option 'A'. Can you explain this answer?

Rahul Kapoor answered
The argument establishes that curiosity is essential for successful learning but acknowledges that most students lack sufficient curiosity initially. So, the missing statement should address how the teacher should approach this situation.
Let's examine each answer choice:
(A) requires for the fulfillment of its goals the stimulation as well as the satisfaction of curiosity
This choice aligns with the argument by stating that the teacher's job requires both stimulating the students' curiosity and providing satisfaction through the learning process. It emphasizes the importance of generating interest and providing gratification for effective learning. This option complements the argument's focus on intrinsic motivation.
(B) necessitates the creative use of rewards that are not inherent in the learning process itself
This choice suggests that the teacher should rely on external rewards to motivate students. However, the argument emphasizes intrinsic gratification rather than extrinsic rewards, making this option less relevant.
(C) is to focus primarily on those topics that do not initially interest the students
This choice proposes that the teacher should focus on topics that initially do not interest students. However, it contradicts the argument's emphasis on curiosity as a prerequisite for successful learning. It suggests avoiding initially interesting topics, which does not align with the argument's logic.
(D) is facilitated by students taking responsibility for their own learning
This choice suggests that students taking responsibility for their learning will facilitate the teacher's job. While student responsibility is essential for effective learning, it does not directly address the issue of insufficient initial curiosity mentioned in the argument.
(E) becomes easier if students realize that some learning is not necessarily enjoyable
This choice implies that accepting the lack of enjoyment in some learning makes the teacher's job easier. However, it does not directly address the challenge of insufficient initial curiosity, which the argument aims to resolve.
Considering the above analysis, (A) is the most logical completion because it aligns with the argument's emphasis on curiosity, stimulation, and satisfaction in the learning process. It acknowledges that the teacher's job involves both stimulating curiosity and providing satisfaction to facilitate successful learning.

Consumer Advocate: The new soft drink, Mango Paradise, contains at least 2.5% of Ephedra, a chemical supplement that stimulates the activity of the nervous system but may also result in serious cardiac problems. Therefore, this drink is harmful to consumer health and should be banned from distribution in our state.
Which of the following is an assumption made by the consumer advocate?
  • a)
    The new soft drink will soon be introduced into mass production.
  • b)
    Consumers are unlikely to enjoy the taste of Mango Paradise because of the high amount of Ephedra contained in the drink.
  • c)
    Any drink that contains at least 2.5% of Ephedra is harmful to consumer health.
  • d)
    The Consumer Advocate is not affiliated with the producer of Mango Paradise.
  • e)
    Most consumers who drink Mango Paradise will eventually experience serious cardiac problems.
Correct answer is option 'C'. Can you explain this answer?

Rahul Kapoor answered
Consumer Advocate: The new soft drink, Mango Paradise, contains at least 2.5% of Ephedra, a chemical supplement that stimulates the activity of the nervous system but may also result in serious cardiac problems. Therefore, this drink is harmful to consumer health and should be banned from distribution in our state.
The consumer advocate is making an argument against the distribution of the soft drink, Mango Paradise, based on the fact that it contains at least 2.5% of Ephedra, a chemical supplement known for its potential to cause serious cardiac problems. In order to support their argument, the consumer advocate is making an assumption. Let's analyze the options:
(a) The new soft drink will soon be introduced into mass production.
This statement does not directly affect the consumer advocate's argument. Whether the soft drink is introduced into mass production or not, if it contains harmful levels of Ephedra, it would still be considered harmful to consumer health.
(b) Consumers are unlikely to enjoy the taste of Mango Paradise because of the high amount of Ephedra contained in the drink.
The consumer advocate's argument is focused on the health risks associated with consuming Ephedra, not the taste preferences of consumers. This statement is not relevant to the argument.
(c) Any drink that contains at least 2.5% of Ephedra is harmful to consumer health.
This assumption aligns directly with the consumer advocate's argument. They are asserting that the soft drink, Mango Paradise, is harmful to consumer health because it contains at least 2.5% of Ephedra. If this assumption is true, it supports the advocate's call for banning the drink.
(d) The Consumer Advocate is not affiliated with the producer of Mango Paradise.
The consumer advocate's affiliation with the producer of Mango Paradise does not directly impact the argument being made. This statement is not relevant to the argument.
(e) Most consumers who drink Mango Paradise will eventually experience serious cardiac problems.
The consumer advocate's argument focuses on the potential for serious cardiac problems due to the presence of Ephedra in Mango Paradise. However, the advocate does not provide evidence or claim that "most consumers" will eventually experience these problems. This statement goes beyond the scope of the argument and is not necessary to support the advocate's position.
Therefore, the assumption made by the consumer advocate is (c) Any drink that contains at least 2.5% of Ephedra is harmful to consumer health. This assumption forms the basis for the advocate's argument against the distribution of Mango Paradise.

Consumer Advocate: Happy Smiles Daycare, a popular child-care facility in Rolling Hills, boasts an average child-to-caregiver ratio of 5:1, a number it cites as the lowest in the county. Furthermore, the daycare claims that compared to some other daycare centers in the county, it does not include helpers, or those who are involved in cleanup and diaper changing, when computing the ratio. Yet Happy Smiles Daycare’s claim that parents with children aged 1–3 will find no other facility with such a low child-to-caregiver ratio is not accurate.
Which of the following, if true, provides the best justification for the Consumer Advocate’s position?
  • a)
    Happy Smiles Daycare has two rooms, one for children 1-2 years old and another for those 2-3 years old, both of which have a child to caregiver ratio of 5:1.
  • b)
    Kenton School, which has fewer than 100 students and a legitimate child-to-caregiver ratio of 6:1, provides approximately equal-sized classes for each year up to age 6, though the classes for those over 3 have a child-to-caregiver ratio more than double that of the other classes.
  • c)
    The number of students enrolled in Happy Smiles Daycare remains relatively fixed throughout the year.
  • d)
    Tiny Tots Daycare, which boasts a 4:1 child-to-caregiver ratio, includes any adults who are in a classroom throughout the day.
  • e)
    Looming budget cuts indicate that Happy Smiles Daycare may not be able to sustain such a low child-to-caregiver ratio in coming years.
Correct answer is option 'B'. Can you explain this answer?

Rahul Kapoor answered
B: Kenton School, which has fewer than 100 students and a legitimate child-to-caregiver ratio of 6:1, provides approximately equal-sized classes for each year up to age 6, though the classes for those over 3 have a child-to-caregiver ratio more than double that of the other classes.
This option provides a counterexample to Happy Smiles Daycare's claim. It states that Kenton School, another daycare facility, has a legitimate child-to-caregiver ratio of 6:1, which is slightly higher than the ratio claimed by Happy Smiles Daycare. Moreover, it mentions that Kenton School provides approximately equal-sized classes for each year up to age 6, indicating that the ratio does not significantly vary based on the age of the children. This suggests that Happy Smiles Daycare's claim of having the lowest child-to-caregiver ratio for children aged 1-3 is inaccurate, as Kenton School offers a similar or lower ratio for children within that age range.

Which of the following most logically completes the argument below?
Nessus Valley Machinery is trying to decide between manufacturing screws out of steel or screws out of aluminum. Both types of screw typically cost about the same to produce and they sell at comparable prices. Because one of Nessus Valley's primary customers, an automotive company, is discontinuing the truck they produce that uses steel screws, the market for steel screws, though not for aluminum screws, will be significantly smaller than usual. Therefore, producing screws from aluminum will be more profitable than producing screws from steel, because __________ .
  • a)
    the smaller-than-usual customer base for steel screws will not be offset by a higher sales price for steel screws
  • b)
    an additional production run of steel screws should be planned for when the market returns to normal levels
  • c)
    the steel screws Nessus Valley produces are better for use in trucks than their aluminum equivalent
  • d)
    aluminum screws can only be used for certain products
  • e)
    producing aluminum screws is more profitable than producing screws from other types of metal, such as titanium
Correct answer is option 'A'. Can you explain this answer?

Rahul Kapoor answered
The argument states that Nessus Valley Machinery is deciding between manufacturing screws out of steel or aluminum. The costs of production and selling prices are comparable for both types of screws. However, due to one of their primary customers discontinuing the use of steel screws in their trucks, the market for steel screws will be significantly smaller than usual, while the market for aluminum screws will remain unaffected.
The argument then concludes that producing screws from aluminum will be more profitable than producing screws from steel. To complete the argument, we need to provide a reason that supports this conclusion.
Let's analyze the answer choices:
(A) The smaller-than-usual customer base for steel screws will not be offset by a higher sales price for steel screws.
This answer choice directly addresses the profitability aspect of the argument. It states that the smaller market for steel screws will not be compensated by a higher sales price for steel screws. This implies that the reduced demand for steel screws will not result in increased profitability, strengthening the case for producing aluminum screws instead.
(B) An additional production run of steel screws should be planned for when the market returns to normal levels.
This answer choice suggests planning an additional production run of steel screws for when the market for them returns to normal levels. While this may be a valid strategy for future operations, it does not directly support the claim that producing aluminum screws will be more profitable in the current situation. Therefore, it is not the most logical completion of the argument.
(C) The steel screws Nessus Valley produces are better for use in trucks than their aluminum equivalent.
This answer choice introduces a new point about the superiority of steel screws for use in trucks compared to aluminum screws. However, this does not directly address the profitability aspect of the argument. The argument is concerned with profitability, not the comparative quality of the screws. Therefore, this answer choice is not the most logical completion of the argument.
(D) Aluminum screws can only be used for certain products.
This answer choice introduces a limitation to the usability of aluminum screws, stating that they can only be used for certain products. While this may be a relevant consideration for the overall product line, it does not provide a direct reason why producing aluminum screws would be more profitable than producing steel screws in the current market situation. Therefore, it is not the most logical completion of the argument.
(E) Producing aluminum screws is more profitable than producing screws from other types of metal, such as titanium.
This answer choice introduces a comparison between aluminum screws and screws made from other types of metal, such as titanium. While this may be interesting information, the argument is specifically focused on deciding between steel and aluminum screws. Therefore, this answer choice is not the most logical completion of the argument.
Based on the analysis above, answer choice (A) is the most logical completion of the argument. It directly addresses the profitability aspect by stating that the reduced demand for steel screws will not be offset by a higher sales price, strengthening the case for producing aluminum screws instead.

Which of the following best completes the passage below?
An Entertainment industry spokesperson's statement that the industry loses millions of dollars every year due to illegal downloads of mobile applications. Yet, this claim is not representative of the truth. Most people who illegally download mobile applications without paying for them are those that have no serious interest in that application. Thus, the loss to the industry is quite small, because......
  • a)
    the total market value of all illegal copies is low in comparison to the total revenue of the entertainment industry
  • b)
    the number of mobile applications that are frequently copied illegally is low in comparison to the number of mobile applications available for sale
  • c)
    most people who illegally copy mobile applications would not purchase them even if purchasing them were the only way to obtain them
  • d)
    even if the entertainment industry received all the revenue it claims to be losing, it would still be experiencing financial difficulties
  • e)
    many users who illegally copy mobile applications never find any use for them
Correct answer is option 'C'. Can you explain this answer?

Rahul Kapoor answered
This question is a completion question, where you need to choose the option that best completes the passage. Let's analyze each option to determine the most suitable choice.
(a) the total market value of all illegal copies is low in comparison to the total revenue of the entertainment industry
This option suggests that the total market value of illegal copies is relatively low compared to the overall revenue generated by the entertainment industry. However, it doesn't directly address the claim that most people who illegally download mobile applications have no serious interest in them. Therefore, it is not the best completion for the passage.
(b) the number of mobile applications that are frequently copied illegally is low in comparison to the number of mobile applications available for sale
This option highlights that the number of mobile applications frequently copied illegally is low compared to the total number of applications available for sale. While this information may be relevant to the topic, it doesn't directly address the claim that most people who engage in illegal downloads have no serious interest in the applications. Hence, it is not the best completion for the passage.
(c) most people who illegally copy mobile applications would not purchase them even if purchasing them were the only way to obtain them
This option directly supports the argument made in the passage. It states that the loss to the entertainment industry is small because most individuals who illegally download mobile applications would not purchase them even if they were unable to obtain them for free. This completion aligns with the statement that those who engage in illegal downloads have no serious interest in the applications. Therefore, option c is the best completion for the passage.
(d) even if the entertainment industry received all the revenue it claims to be losing, it would still be experiencing financial difficulties
This option introduces a new concept by suggesting that even if the industry received all the revenue it claims to be losing, it would still face financial difficulties. While this information may be relevant to the topic of the entertainment industry's financial situation, it does not directly address the claim that most people who illegally download mobile applications have no serious interest in them. Hence, it is not the best completion for the passage.
(e) many users who illegally copy mobile applications never find any use for them
This option suggests that many users who engage in illegal downloads never find any use for the applications they obtain. Although this information is relevant, it does not directly support the claim that most people who engage in illegal downloads have no serious interest in the applications. Therefore, it is not the best completion for the passage.
In conclusion, the option that best completes the passage is (c) most people who illegally copy mobile applications would not purchase them even if purchasing them were the only way to obtain them.

Anatomical bilateral symmetry is a common trait. It follows, therefore, that it confers survival advantages on organisms. After all, if bilateral symmetry did not confer such advantages, it would not be common.

The pattern of reasoning in which one of the following arguments is most similar to that in the argument above?
  • a)
    Since it is Sawyer who is negotiating for the city government, it must be true that the city takes the matter seriously. After all, if Sawyer had not been available, the city would have insisted that the negotiations be deferred.
  • b)
    Clearly, no candidate is better qualified for the job than Trumbull. In fact, even to suggest that there might be a more highly qualified candidate seems absurd to those who have seen Trumbull at work.
  • c)
    If Powell lacked superior negotiating skills, she would not have been appointed arbitrator in this case. As everyone knows, she is the appointed arbitrator, so her negotiating skills are, detractors notwithstanding, bound to be superior.
  • d)
    Since Varga was away on vacation at the time, it must have been Rivers who conducted the secret negotiations. Any other scenario makes little sense, for Rivers never does the negotiating unless Varga is unavailable.
  • e)
    If Wong is appointed arbitrator, a decision will be reached promptly. Since it would be absurd to appoint anyone other than Wong as arbitrator, a prompt decision can reasonably be expected.
Correct answer is option 'C'. Can you explain this answer?

Rahul Kapoor answered
(C) If Powell lacked superior negotiating skills, she would not have been appointed arbitrator in this case. As everyone knows, she is the appointed arbitrator, so her negotiating skills are, detractors notwithstanding, bound to be superior.
In both the original argument and option (C), a conclusion is drawn based on the assumption that a certain condition or quality must exist because if it did not, a specific outcome or situation would not occur. The original argument assumes that if bilateral symmetry did not confer survival advantages, it would not be a common trait. Similarly, option (C) assumes that if Powell lacked superior negotiating skills, she would not have been appointed as an arbitrator in the case. Both arguments rely on the idea that the existence of a certain outcome implies the existence of a particular condition or quality.

From the fact that people who studied music as children frequently are quite proficient at mathematics, it cannot be concluded that the skills required for mathematics are acquired by studying music: it is equally likely that proficiency in mathematics and studying music are both the result of growing up in a family that encourages its children to excel at all intellectual and artistic endeavors.
The pattern of reasoning in which one of the following arguments is most parallel to that in the argument above?
  • a)
    Although children who fail to pay attention tend to perform poorly in school, it should not necessarily be thought that their poor performance is caused by their failure to pay attention, for it is always possible that their failure to pay attention is due to undiagnosed hearing problems that can also lead to poor performance in school.
  • b)
    People who attend a university in a foreign country are usually among the top students from their native country. It would therefore be wrong to conclude from the fact that many foreign students perform better academically than others in this country that secondary schools in other countries are superior to those in this country; it may be that evaluation standards are different.
  • c)
    People whose diet includes relatively large quantities of certain fruits and vegetables have a slightly lower than average incidence of heart disease. But it would be premature to conclude that consuming these fruits and vegetables prevents heart disease, for this correlation may be merely coincidental.
  • d)
    Those who apply to medical school are required to study biology and chemistry. It would be a mistake, however, to conclude that those who have mastered chemistry and biology will succeed as physicians, for the practical application of knowledge is different from its acquisition.
  • e)
    Those who engage in vigorous exercise tend to be very healthy. But it would be silly to conclude that vigorous exercise is healthful simply because people who are healthy exercise vigorously, since it is possible that exercise that is less vigorous also has beneficial results.
Correct answer is option 'A'. Can you explain this answer?

Rahul Kapoor answered
Answer choice (A): This option is the correct one. Similar to the information provided, there is a correlation between lack of attention and poor academic performance. Both arguments caution against assuming a cause-and-effect relationship, as there could be a third factor (such as undiagnosed hearing problems) that is responsible for both.
Answer choice (B): This answer choice is incorrect because it does not consider the possibility of a common cause for the correlated events.
Answer choice (C): Although this option questions the validity of a causal relationship, like the stimulus, it fails to speculate on a common cause that could explain the correlation. Since this speculation was essential in the stimulus argument, this answer choice is incorrect.
Answer choice (D): This answer choice simply suggests that studying biology and chemistry alone is not sufficient for success as a physician.
Answer choice (E): Initially, this option may seem attractive as vigorous exercise does not guarantee good health, just as studying music does not guarantee improved math skills. However, the parallel ends there. There is no reason to suspect a common cause explaining the correlation between vigorous exercise and health. Instead, we are informed that vigorous exercise may not be essential, as less intense exercise could also have beneficial effects. This is similar to saying that studying music does not necessarily enhance math skills because listening to music alone might be sufficient. Therefore, this answer choice is incorrect.

In a recent study, Mario García argues that in the United States between 1930 and 1960 the group of political activists he calls the “Mexican American Generation” was more radical and politically diverse (5) than earlier historians have recognized. Through analysis of the work of some of the era’s most important scholars, García does provide persuasive evidence that in the 1930s and 1940s these activists anticipated many of the reforms proposed by the more (10) militant Chicanos of the 1960s and 1970s. His study, however, suffers from two flaws.
First, García’s analysis of the evidence he provides to demonstrate the Mexican American Generation’s political diversity is not entirely (15) consistent. Indeed, he undermines his primary thesis by emphasizing an underlying consensus among various groups that tends to conceal the full significance of their differences. Groups such as the League of United Latin American Citizens, an (20) organization that encouraged Mexican Americans to pursue a civil rights strategy of assimilation into the United States political and cultural mainstream, were often diametrically opposed to organizations such as the Congress of Spanish-Speaking People, a coalition (25) group that advocated bilingual education and equal rights for resident aliens in the United States. García acknowledges these differences but dismisses them as insignificant, given that the goals of groups as disparate as these centered on liberal reform, not (30) revolution. But one need only note the fierce controversies that occurred during the period over United States immigration policies and the question of assimilation versus cultural maintenance to recognize that Mexican American political history since 1930 (35) has been characterized not by consensus but by intense and lively debate.
Second, García may be exaggerating the degree to which the views of these activists were representative of the ethnic Mexican population residing in the (40) United States during this period. Noting that by 1930 the proportion of the Mexican American population that had been born in the United States had significantly increased, García argues that between 1930 and 1960 a new generation of Mexican American (45) leaders appeared, one that was more acculturated and hence more politically active than its predecessor. Influenced by their experience of discrimination and by the inclusive rhetoric of World War II slogans, these leaders, according to García, were determined to (50) achieve full civil rights for all United States residents of Mexican descent. However, it is not clear how far this outlook extended beyond these activists. Without a better understanding of the political implications of important variables such as patterns of bilingualism (55) and rates of Mexican immigration and naturalization, and the variations in ethnic consciousness these variables help to create, one cannot assume that an increase in the proportion of Mexican Americans born in the United States necessarily resulted in an increase (60) in the ethnic Mexican population’s political activism.
It can be inferred that the author of the passage believes which one of the following about the Mexican American political activists of the 1930s and 1940s?
  • a)
    Their common goal of liberal reform made them less militant than the Mexican American activists of the 1960s and 1970s.
  • b)
    Their common goal of liberal reform did not outweigh their political differences.
  • c)
    Their common goal of liberal reform helped them reach a consensus in spite of their political differences.
  • d)
    They were more or less evenly divided between those favoring assimilation and those favoring cultural maintenance.
  • e)
    They did not succeed in fully achieving their political goals because of their disparate political views.
Correct answer is option 'B'. Can you explain this answer?

Wizius Careers answered
To determine which statement can be inferred about the Mexican American political activists of the 1930s and 1940s, let’s analyze the passage’s content and the options provided:
  1. Option A: "Their common goal of liberal reform made them less militant than the Mexican American activists of the 1960s and 1970s."
    • The passage does not specifically discuss the militancy of activists from the 1930s and 1940s compared to those of the 1960s and 1970s. It focuses more on the diversity and political differences among groups, not directly on their relative levels of militancy.
  2. Option B: "Their common goal of liberal reform did not outweigh their political differences."
    • The passage highlights that despite a common goal of liberal reform, there were significant political differences among the groups. García’s study is critiqued for not fully addressing these differences and for emphasizing a consensus that might obscure the diversity of opinions. This suggests that their shared goals did not negate the substantial political disagreements.
  3. Option C: "Their common goal of liberal reform helped them reach a consensus in spite of their political differences."
    • The passage argues that the emphasis on a consensus might obscure the true political diversity and disagreements among the groups. It implies that the activists’ shared goals did not necessarily lead to consensus or diminish their political differences.
  4. Option D: "They were more or less evenly divided between those favoring assimilation and those favoring cultural maintenance."
    • The passage does not provide specific information on how evenly divided the activists were between assimilation and cultural maintenance. It only mentions that there were significant differences between groups with opposing views.
  5. Option E: "They did not succeed in fully achieving their political goals because of their disparate political views."
    • The passage does not address the success or failure of the activists in achieving their goals. It focuses on the differences in political views rather than on their outcomes.
Correct Option: B
Explanation: The passage critiques García’s study for highlighting an underlying consensus among the groups, which conceals their significant political differences. This suggests that while they shared a common goal of liberal reform, their political differences remained substantial and were not outweighed by their shared objectives. Thus, Option B is the best inference based on the passage.

Experts anticipate that global atmospheric concentrations of carbon dioxide (CO2) will have doubled by the end of the twenty-first century. It is known that CO2 can contribute to global warming by (5) trapping solar energy that is being reradiated as heat from the Earth’s surface. However, some research has suggested that elevated CO2 levels could enhance the photosynthetic rates of plants, resulting in a lush world of agricultural abundance, and that this CO2 (10) fertilization effect might eventually decrease the rate of global warming. The increased vegetation in such an environment could be counted on to draw more CO2 from the atmosphere. The level of CO2 would thus increase at a lower rate than many experts have (15) predicted.
However, while a number of recent studies confirm that plant growth would be generally enhanced in an atmosphere rich in CO2, they also suggest that increased CO2 would differentially increase the growth (20) rate of different species of plants, which could eventually result in decreased agricultural yields. Certain important crops such as corn and sugarcane that currently have higher photosynthetic efficiencies than other plants may lose that edge in an atmosphere (25) rich in CO2. Patterson and Flint have shown that these important crops may experience yield reductions because of the increased performance of certain weeds. Such differences in growth rates between plant species could also alter ecosystem stability. Studies have (30) shown that within rangeland regions, for example, a weedy grass grows much better with plentiful CO2 than do three other grasses. Because this weedy grass predisposes land to burning, its potential increase may lead to greater numbers of and more severe wildfires in (35) future rangeland communities.
It is clear that the CO2 fertilization effect does not guarantee the lush world of agricultural abundance that once seemed likely, but what about the potential for the increased uptake of CO2 to decrease the rate of global (40) warming? Some studies suggest that the changes accompanying global warming will not improve the ability of terrestrial ecosystems to absorb CO2. Billings’ simulation of global warming conditions in wet tundra grasslands showed that the level of CO2 (45) actually increased. Plant growth did increase under these conditions because of warmer temperatures and increased CO2 levels. But as the permafrost melted, more peat (accumulated dead plant material) began to decompose. This process in turn liberated more CO2 to (50) the atmosphere. Billings estimated that if summer temperatures rose four degrees Celsius, the tundra would liberate 50 percent more CO2 than it does currently. In a warmer world, increased plant growth, which could absorb CO2 from the atmosphere, would (55) not compensate for this rapid increase in decomposition rates. This observation is particularly important because high-latitude habitats such as the tundra are expected to experience the greatest temperature increase.
Which one of the following best states the main point of the passage?
  • a)
    Elevated levels of CO2 would enhance photosynthetic rates, thus increasing plant growth and agricultural yields.
  • b)
    Recent studies have yielded contradictory findings about the benefits of increased levels of CO2 on agricultural productivity.
  • c)
    The possible beneficial effects of increased levels of CO2 on plant growth and global warming have been overstated.
  • d)
    Increased levels of CO2 would enhance the growth rates of certain plants, but would inhibit the growth rates of other plants.
  • e)
    Increased levels of CO2 would increase plant growth, but the rate of global warming would ultimately increase.
Correct answer is option 'C'. Can you explain this answer?

Wizius Careers answered
 The passage discusses how increased CO2 levels might initially seem beneficial for plant growth and potentially mitigating global warming. However, it then goes on to explain the complexities and negative side effects, suggesting that the benefits have been overstated. Thus, option (C) best captures the main point of the passage.

Passage A is from a source published in 2004 and passage B is from a source published in 2007.
Passage A
Millions of people worldwide play multiplayer online games. They each pick, say, a medieval character to play, such as a warrior. Then they might band together in quests to slay magical beasts; their (5) avatars appear as tiny characters striding across a Tolkienesque land.
The economist Edward Castronova noticed something curious about the game he played: it had its own economy, a bustling trade in virtual goods. (10) Players generate goods as they play, often by killing creatures for their treasure and trading it. The longer they play, the wealthier they get.
Things got even more interesting when Castronova learned about the “player auctions.” Players would (15) sometimes tire of the game and decide to sell off their virtual possessions at online auction sites.
As Castronova stared at the auction listings, he recognized with a shock what he was looking at. It was a form of currency trading! Each item had a value (20) in the virtual currency traded in the game; when it was sold on the auction site, someone was paying cold hard cash for it. That meant that the virtual currency was worth something in real currency. Moreover, since players were killing monsters or skinning animals to (25) sell their pelts, they were, in effect, creating wealth.
Passage B
Most multiplayer online games prohibit real-world trade in virtual items, but some actually encourage it, for example, by granting participants intellectual property rights in their creations.
(30) Although it seems intuitively the case that someone who accepts real money for the transfer of a virtual item should be taxed, what about the player who only accumulates items or virtual currency within a virtual world? Is “loot” acquired in a game taxable, (35) as a prize or award is? And is the profit in a purely in-game trade or sale for virtual currency taxable? These are important questions, given the tax revenues at stake, and there is pressure on governments to answer them, given that the economies of some virtual (40) worlds are comparable to those of small countries.
Most people’s intuition probably would be that accumulation of assets within a game should not be taxed even though income tax applies even to noncash accessions to wealth. This article will argue that (45) income tax law and policy support that result. Loot acquisitions in game worlds should not be treated as taxable prizes and awards, but rather should be treated like other property that requires effort to obtain, such as fish pulled from the ocean, which is taxed only (50) upon sale. Moreover, in-game trades of virtual items should not be treated as taxable barter.
By contrast, tax doctrine and policy counsel taxation of the sale of virtual items for real currency, and, in games that are intentionally commodified, (55) even of in-world sales for virtual currency, regardless of whether the participant cashes out. This approach would leave entertainment value untaxed without creating a tax shelter for virtual commerce.
Based on what can be inferred from their titles, the relationship between which one of the following pairs of documents is most analogous to the relationship between passage A and passage B?
  • a)
    “Advances in Artificial Intelligence” “Human Psychology Applied to Robots”
  • b)
    “Internet Retailers Post Good Year” “Lawmakers Move to Tax Internet Commerce”
  • c)
    “New Planet Discovered in Solar System” “Planet or Asteroid: Scientists Debate”
  • d)
    “Biologists Create New Species in Lab” “Artificially Created Life: How Patent Law Applies”
  • e)
    “A Renegade Economist’s Views on Taxation” “Candidate Runs on Unorthodox Tax Plan”
Correct answer is option 'D'. Can you explain this answer?

To determine the most analogous relationship between the pairs of documents, let’s examine how the titles of Passage A and Passage B relate to each other:
Passage A:
  • Content: Discusses the discovery of a virtual economy in online games and the surprising realization of real-world value for virtual items and currency.
  • Focus: The focus is on the novel economic phenomenon and the excitement around the discovery of virtual goods having real-world economic significance.
Passage B:
  • Content: Addresses the legal and taxation issues related to virtual items, focusing on how to handle the taxation of virtual goods and currency.
  • Focus: The focus is on the legal implications and policy discussions regarding virtual economies and their taxation.
Analogy Analysis:
  1. (A) “Advances in Artificial Intelligence” “Human Psychology Applied to Robots”
    • This pair discusses advances in technology and its application, which is more about the technological implications rather than a novel discovery followed by its policy implications.
  2. (B) “Internet Retailers Post Good Year” “Lawmakers Move to Tax Internet Commerce”
    • This pair reflects the relationship between economic success and regulatory responses, which parallels how Passage A’s discovery of virtual economies relates to Passage B’s discussion of taxation.
  3. (C) “New Planet Discovered in Solar System” “Planet or Asteroid: Scientists Debate”
    • This pair shows a discovery followed by debates on classification, which is similar to discovering a new economic phenomenon and then discussing its policy implications.
  4. (D) “Biologists Create New Species in Lab” “Artificially Created Life: How Patent Law Applies”
    • This pair involves a novel scientific achievement (creation of a new species) followed by legal considerations (patent law), which closely mirrors the structure of Passage A’s economic discovery and Passage B’s legal discussion.
  5. (E) “A Renegade Economist’s Views on Taxation” “Candidate Runs on Unorthodox Tax Plan”
    • This pair features views and proposals related to taxation, not a novel discovery followed by its policy implications.
Conclusion: The most analogous relationship is between:
(D) “Biologists Create New Species in Lab” “Artificially Created Life: How Patent Law Applies”
This pair mirrors the relationship between Passage A and Passage B, where Passage A represents a novel discovery and Passage B deals with the subsequent policy or legal implications of that discovery.

Over the past 50 years, expansive, low-density communities have proliferated at the edges of many cities in the United States and Canada, creating a phenomenon known as suburban sprawl. Andres (5) Duany, Elizabeth Plater-Zyberk, and Jeff Speck, a group of prominent town planners belonging to a movement called New Urbanism, contend that suburban sprawl contributes to the decline of civic life and civility. For reasons involving the flow of (10) automobile traffic, they note, zoning laws usually dictate that suburban homes, stores, businesses, and schools be built in separate areas, and this separation robs people of communal space where they can interact and get to know one another. It is as difficult (15) to imagine the concept of community without a town square or local pub, these town planners contend, as it is to imagine the concept of family independent of the home.
Suburban housing subdivisions, Duany and his (20) colleagues add, usually contain homes identical not only in appearance but also in price, resulting in a de facto economic segregation of residential neighborhoods. Children growing up in these neighborhoods, whatever their economic (25) circumstances, are certain to be ill prepared for life in a diverse society. Moreover, because the widely separated suburban homes and businesses are connected only by “collector roads,” residents are forced to drive, often in heavy traffic, in order to (30) perform many daily tasks. Time that would in a town center involve social interaction within a physical public realm is now spent inside the automobile, where people cease to be community members and instead become motorists, competing for road space, (35) often acting antisocially. Pedestrians rarely act in this manner toward each other. Duany and his colleagues advocate development based on early-twentieth- century urban neighborhoods that mix housing of different prices and offer residents a “gratifying (40) public realm” that includes narrow, tree-lined streets, parks, corner grocery stores, cafes, small neighborhood schools, all within walking distance. This, they believe, would give people of diverse backgrounds and lifestyles an opportunity to interact (45) and thus develop mutual respect.
Opponents of New Urbanism claim that migration to sprawling suburbs is an expression of people’s legitimate desire to secure the enjoyment and personal mobility provided by the automobile and the (50) lifestyle that it makes possible. However, the New Urbanists do not question people’s right to their own values; instead, they suggest that we should take a more critical view of these values and of the sprawl conducive zoning and subdivision policies that reflect (55) them. New Urbanists are fundamentally concerned with the long-term social costs of the now-prevailing attitude that individual mobility, consumption, and wealth should be valued absolutely, regardless of their impact on community life.
Which one of the following, if true, would most weaken the position that the passage attributes to critics of the New Urbanists?
  • a)
    Most people who spend more time than they would like getting from one daily task to another live in central areas of large cities.
  • b)
    Most people who often drive long distances for shopping and entertainment live in small towns rather than in suburban areas surrounding large cities.
  • c)
    Most people who have easy access to shopping and entertainment do not live in suburban areas.
  • d)
    Most people who choose to live in sprawling suburbs do so because comparable housing in neighborhoods that do not require extensive automobile travel is more expensive.
  • e)
    Most people who vote in municipal elections do not cast their votes on the basis of candidates’ positions on zoning policies.
Correct answer is option 'D'. Can you explain this answer?

Wizius Careers answered
Critics of New Urbanists argue that people migrate to suburbs to enjoy the mobility and lifestyle enabled by the automobile. If it is true that people live in sprawling suburbs because housing in more accessible neighborhoods is more expensive, it implies that economic factors, not just lifestyle preferences, drive suburban living. This weakens the critics' position by showing that the choice to live in suburbs is influenced by affordability rather than a pure preference for automobile-dependent living.

Chapter doubts & questions for Verbal Reasoning (Level-wise Complete Tests) - Practice Questions for GMAT 2025 is part of GMAT exam preparation. The chapters have been prepared according to the GMAT exam syllabus. The Chapter doubts & questions, notes, tests & MCQs are made for GMAT 2025 Exam. Find important definitions, questions, notes, meanings, examples, exercises, MCQs and online tests here.

Chapter doubts & questions of Verbal Reasoning (Level-wise Complete Tests) - Practice Questions for GMAT in English & Hindi are available as part of GMAT exam. Download more important topics, notes, lectures and mock test series for GMAT Exam by signing up for free.

Top Courses GMAT